Your favourite application of the Baire Category Theorem












180














I think I remember reading somewhere that the Baire Category Theorem is supposedly quite powerful. Whether that is true or not, it's my favourite theorem (so far) and I'd love to see some applications that confirm its neatness and/or power.



Here's the theorem (with proof) and two applications:





(Baire) A non-empty complete metric space $X$ is not a countable union of nowhere dense sets.



Proof: Let $X = bigcup U_i$ where $mathring{overline{U_i}} = varnothing$. We construct a Cauchy sequence as follows: Let $x_1$ be any point in $(overline{U_1})^c$. We can find such a point because $(overline{U_1})^c subset X$ and $X$ contains at least one non-empty open set (if nothing else, itself) but $mathring{overline{U_1}} = varnothing$ which is the same as saying that $overline{U_1}$ does not contain any open sets hence the open set contained in $X$ is contained in $overline{U_1}^c$. Hence we can pick $x_1$ and $varepsilon_1 > 0$ such that $B(x_1, varepsilon_1) subset (overline{U_1})^c subset U_1^c$.



Next we make a similar observation about $U_2$ so that we can find $x_2$ and $varepsilon_2 > 0$ such that $B(x_2, varepsilon_2) subset overline{U_2}^c cap B(x_1, frac{varepsilon_1}{2})$. We repeat this process to get a sequence of balls such that $B_{k+1} subset B_k$ and a sequence $(x_k)$ that is Cauchy. By completeness of $X$, $lim x_k =: x$ is in $X$. But $x$ is in $B_k$ for every $k$ hence not in any of the $U_i$ and hence not in $bigcup U_i = X$. Contradiction. $Box$





Here is one application (taken from here):



Claim: $[0,1]$ contains uncountably many elements.



Proof: Assume that it contains countably many. Then $[0,1] = bigcup_{x in (0,1)} {x}$ and since ${x}$ are nowhere dense sets, $X$ is a countable union of nowhere dense sets. But $[0,1]$ is complete, so we have a contradiction. Hence $X$ has to be uncountable.





And here is another one (taken from here):



Claim: The linear space of all polynomials in one variable is not a Banach space in any norm.



Proof: "The subspace of polynomials of degree $leq n$ is closed in any norm because it is finite-dimensional. Hence the space of all polynomials can be written as countable union of closed nowhere dense sets. If there were a complete norm this would contradict the Baire Category Theorem."










share|cite|improve this question




















  • 3




    mathoverflow.net/questions/34059/…
    – user38268
    Jul 2 '12 at 13:34






  • 2




    mathoverflow.net/questions/56323/… and math.stackexchange.com/questions/135947/…
    – Eugene
    Jul 2 '12 at 13:39








  • 1




    In the proof that $[0,1]$ is uncountable you mean to say that it is the union of $[0,1]$ and not $(0,1)$. Otherwise you are missing two points.
    – Asaf Karagila
    Jul 2 '12 at 22:27






  • 4




    A nonempty complete metric space $X$ is not a countable union of nowhere dense sets.
    – nullUser
    Jul 3 '12 at 1:50






  • 1




    A nice survey paper: MR1640007 (99h:26012). Jones, Sara Hawtrey. Applications of the Baire category theorem. Real Anal. Exchange 23 (2), (1997/98), 363–394. It should be available through Project Euclid.
    – Andrés E. Caicedo
    Apr 12 '13 at 19:18
















180














I think I remember reading somewhere that the Baire Category Theorem is supposedly quite powerful. Whether that is true or not, it's my favourite theorem (so far) and I'd love to see some applications that confirm its neatness and/or power.



Here's the theorem (with proof) and two applications:





(Baire) A non-empty complete metric space $X$ is not a countable union of nowhere dense sets.



Proof: Let $X = bigcup U_i$ where $mathring{overline{U_i}} = varnothing$. We construct a Cauchy sequence as follows: Let $x_1$ be any point in $(overline{U_1})^c$. We can find such a point because $(overline{U_1})^c subset X$ and $X$ contains at least one non-empty open set (if nothing else, itself) but $mathring{overline{U_1}} = varnothing$ which is the same as saying that $overline{U_1}$ does not contain any open sets hence the open set contained in $X$ is contained in $overline{U_1}^c$. Hence we can pick $x_1$ and $varepsilon_1 > 0$ such that $B(x_1, varepsilon_1) subset (overline{U_1})^c subset U_1^c$.



Next we make a similar observation about $U_2$ so that we can find $x_2$ and $varepsilon_2 > 0$ such that $B(x_2, varepsilon_2) subset overline{U_2}^c cap B(x_1, frac{varepsilon_1}{2})$. We repeat this process to get a sequence of balls such that $B_{k+1} subset B_k$ and a sequence $(x_k)$ that is Cauchy. By completeness of $X$, $lim x_k =: x$ is in $X$. But $x$ is in $B_k$ for every $k$ hence not in any of the $U_i$ and hence not in $bigcup U_i = X$. Contradiction. $Box$





Here is one application (taken from here):



Claim: $[0,1]$ contains uncountably many elements.



Proof: Assume that it contains countably many. Then $[0,1] = bigcup_{x in (0,1)} {x}$ and since ${x}$ are nowhere dense sets, $X$ is a countable union of nowhere dense sets. But $[0,1]$ is complete, so we have a contradiction. Hence $X$ has to be uncountable.





And here is another one (taken from here):



Claim: The linear space of all polynomials in one variable is not a Banach space in any norm.



Proof: "The subspace of polynomials of degree $leq n$ is closed in any norm because it is finite-dimensional. Hence the space of all polynomials can be written as countable union of closed nowhere dense sets. If there were a complete norm this would contradict the Baire Category Theorem."










share|cite|improve this question




















  • 3




    mathoverflow.net/questions/34059/…
    – user38268
    Jul 2 '12 at 13:34






  • 2




    mathoverflow.net/questions/56323/… and math.stackexchange.com/questions/135947/…
    – Eugene
    Jul 2 '12 at 13:39








  • 1




    In the proof that $[0,1]$ is uncountable you mean to say that it is the union of $[0,1]$ and not $(0,1)$. Otherwise you are missing two points.
    – Asaf Karagila
    Jul 2 '12 at 22:27






  • 4




    A nonempty complete metric space $X$ is not a countable union of nowhere dense sets.
    – nullUser
    Jul 3 '12 at 1:50






  • 1




    A nice survey paper: MR1640007 (99h:26012). Jones, Sara Hawtrey. Applications of the Baire category theorem. Real Anal. Exchange 23 (2), (1997/98), 363–394. It should be available through Project Euclid.
    – Andrés E. Caicedo
    Apr 12 '13 at 19:18














180












180








180


152





I think I remember reading somewhere that the Baire Category Theorem is supposedly quite powerful. Whether that is true or not, it's my favourite theorem (so far) and I'd love to see some applications that confirm its neatness and/or power.



Here's the theorem (with proof) and two applications:





(Baire) A non-empty complete metric space $X$ is not a countable union of nowhere dense sets.



Proof: Let $X = bigcup U_i$ where $mathring{overline{U_i}} = varnothing$. We construct a Cauchy sequence as follows: Let $x_1$ be any point in $(overline{U_1})^c$. We can find such a point because $(overline{U_1})^c subset X$ and $X$ contains at least one non-empty open set (if nothing else, itself) but $mathring{overline{U_1}} = varnothing$ which is the same as saying that $overline{U_1}$ does not contain any open sets hence the open set contained in $X$ is contained in $overline{U_1}^c$. Hence we can pick $x_1$ and $varepsilon_1 > 0$ such that $B(x_1, varepsilon_1) subset (overline{U_1})^c subset U_1^c$.



Next we make a similar observation about $U_2$ so that we can find $x_2$ and $varepsilon_2 > 0$ such that $B(x_2, varepsilon_2) subset overline{U_2}^c cap B(x_1, frac{varepsilon_1}{2})$. We repeat this process to get a sequence of balls such that $B_{k+1} subset B_k$ and a sequence $(x_k)$ that is Cauchy. By completeness of $X$, $lim x_k =: x$ is in $X$. But $x$ is in $B_k$ for every $k$ hence not in any of the $U_i$ and hence not in $bigcup U_i = X$. Contradiction. $Box$





Here is one application (taken from here):



Claim: $[0,1]$ contains uncountably many elements.



Proof: Assume that it contains countably many. Then $[0,1] = bigcup_{x in (0,1)} {x}$ and since ${x}$ are nowhere dense sets, $X$ is a countable union of nowhere dense sets. But $[0,1]$ is complete, so we have a contradiction. Hence $X$ has to be uncountable.





And here is another one (taken from here):



Claim: The linear space of all polynomials in one variable is not a Banach space in any norm.



Proof: "The subspace of polynomials of degree $leq n$ is closed in any norm because it is finite-dimensional. Hence the space of all polynomials can be written as countable union of closed nowhere dense sets. If there were a complete norm this would contradict the Baire Category Theorem."










share|cite|improve this question















I think I remember reading somewhere that the Baire Category Theorem is supposedly quite powerful. Whether that is true or not, it's my favourite theorem (so far) and I'd love to see some applications that confirm its neatness and/or power.



Here's the theorem (with proof) and two applications:





(Baire) A non-empty complete metric space $X$ is not a countable union of nowhere dense sets.



Proof: Let $X = bigcup U_i$ where $mathring{overline{U_i}} = varnothing$. We construct a Cauchy sequence as follows: Let $x_1$ be any point in $(overline{U_1})^c$. We can find such a point because $(overline{U_1})^c subset X$ and $X$ contains at least one non-empty open set (if nothing else, itself) but $mathring{overline{U_1}} = varnothing$ which is the same as saying that $overline{U_1}$ does not contain any open sets hence the open set contained in $X$ is contained in $overline{U_1}^c$. Hence we can pick $x_1$ and $varepsilon_1 > 0$ such that $B(x_1, varepsilon_1) subset (overline{U_1})^c subset U_1^c$.



Next we make a similar observation about $U_2$ so that we can find $x_2$ and $varepsilon_2 > 0$ such that $B(x_2, varepsilon_2) subset overline{U_2}^c cap B(x_1, frac{varepsilon_1}{2})$. We repeat this process to get a sequence of balls such that $B_{k+1} subset B_k$ and a sequence $(x_k)$ that is Cauchy. By completeness of $X$, $lim x_k =: x$ is in $X$. But $x$ is in $B_k$ for every $k$ hence not in any of the $U_i$ and hence not in $bigcup U_i = X$. Contradiction. $Box$





Here is one application (taken from here):



Claim: $[0,1]$ contains uncountably many elements.



Proof: Assume that it contains countably many. Then $[0,1] = bigcup_{x in (0,1)} {x}$ and since ${x}$ are nowhere dense sets, $X$ is a countable union of nowhere dense sets. But $[0,1]$ is complete, so we have a contradiction. Hence $X$ has to be uncountable.





And here is another one (taken from here):



Claim: The linear space of all polynomials in one variable is not a Banach space in any norm.



Proof: "The subspace of polynomials of degree $leq n$ is closed in any norm because it is finite-dimensional. Hence the space of all polynomials can be written as countable union of closed nowhere dense sets. If there were a complete norm this would contradict the Baire Category Theorem."







general-topology functional-analysis big-list baire-category






share|cite|improve this question















share|cite|improve this question













share|cite|improve this question




share|cite|improve this question








edited Apr 13 '17 at 12:21


























community wiki





6 revs, 3 users 50%
Rudy the Reindeer









  • 3




    mathoverflow.net/questions/34059/…
    – user38268
    Jul 2 '12 at 13:34






  • 2




    mathoverflow.net/questions/56323/… and math.stackexchange.com/questions/135947/…
    – Eugene
    Jul 2 '12 at 13:39








  • 1




    In the proof that $[0,1]$ is uncountable you mean to say that it is the union of $[0,1]$ and not $(0,1)$. Otherwise you are missing two points.
    – Asaf Karagila
    Jul 2 '12 at 22:27






  • 4




    A nonempty complete metric space $X$ is not a countable union of nowhere dense sets.
    – nullUser
    Jul 3 '12 at 1:50






  • 1




    A nice survey paper: MR1640007 (99h:26012). Jones, Sara Hawtrey. Applications of the Baire category theorem. Real Anal. Exchange 23 (2), (1997/98), 363–394. It should be available through Project Euclid.
    – Andrés E. Caicedo
    Apr 12 '13 at 19:18














  • 3




    mathoverflow.net/questions/34059/…
    – user38268
    Jul 2 '12 at 13:34






  • 2




    mathoverflow.net/questions/56323/… and math.stackexchange.com/questions/135947/…
    – Eugene
    Jul 2 '12 at 13:39








  • 1




    In the proof that $[0,1]$ is uncountable you mean to say that it is the union of $[0,1]$ and not $(0,1)$. Otherwise you are missing two points.
    – Asaf Karagila
    Jul 2 '12 at 22:27






  • 4




    A nonempty complete metric space $X$ is not a countable union of nowhere dense sets.
    – nullUser
    Jul 3 '12 at 1:50






  • 1




    A nice survey paper: MR1640007 (99h:26012). Jones, Sara Hawtrey. Applications of the Baire category theorem. Real Anal. Exchange 23 (2), (1997/98), 363–394. It should be available through Project Euclid.
    – Andrés E. Caicedo
    Apr 12 '13 at 19:18








3




3




mathoverflow.net/questions/34059/…
– user38268
Jul 2 '12 at 13:34




mathoverflow.net/questions/34059/…
– user38268
Jul 2 '12 at 13:34




2




2




mathoverflow.net/questions/56323/… and math.stackexchange.com/questions/135947/…
– Eugene
Jul 2 '12 at 13:39






mathoverflow.net/questions/56323/… and math.stackexchange.com/questions/135947/…
– Eugene
Jul 2 '12 at 13:39






1




1




In the proof that $[0,1]$ is uncountable you mean to say that it is the union of $[0,1]$ and not $(0,1)$. Otherwise you are missing two points.
– Asaf Karagila
Jul 2 '12 at 22:27




In the proof that $[0,1]$ is uncountable you mean to say that it is the union of $[0,1]$ and not $(0,1)$. Otherwise you are missing two points.
– Asaf Karagila
Jul 2 '12 at 22:27




4




4




A nonempty complete metric space $X$ is not a countable union of nowhere dense sets.
– nullUser
Jul 3 '12 at 1:50




A nonempty complete metric space $X$ is not a countable union of nowhere dense sets.
– nullUser
Jul 3 '12 at 1:50




1




1




A nice survey paper: MR1640007 (99h:26012). Jones, Sara Hawtrey. Applications of the Baire category theorem. Real Anal. Exchange 23 (2), (1997/98), 363–394. It should be available through Project Euclid.
– Andrés E. Caicedo
Apr 12 '13 at 19:18




A nice survey paper: MR1640007 (99h:26012). Jones, Sara Hawtrey. Applications of the Baire category theorem. Real Anal. Exchange 23 (2), (1997/98), 363–394. It should be available through Project Euclid.
– Andrés E. Caicedo
Apr 12 '13 at 19:18










24 Answers
24






active

oldest

votes


















50














If $P$ is an infinitely differentiable function such that for each $x$, there is an $n$ with $P^{(n)}(x)=0$, then $P$ is a polynomial. (Note $n$ depends on $x$.) See the discussion in Math Overflow.






share|cite|improve this answer



















  • 2




    Link, taken from comments: mathoverflow.net/questions/34059/…
    – sdcvvc
    Jul 4 '12 at 12:03



















39














The uniform boundedness principle of Functional Analysis is a very important application of the Baire Category Theorem.



Added: (t.b.) See also Sokal's A really simple elementary proof of the uniform boundedness theorem for a proof without Baire.






share|cite|improve this answer



















  • 3




    The other one of the big theorems from beginning functional analysis that uses the Baire Category Theorem is the open mapping theorem
    – Francis Adams
    Jul 2 '12 at 14:05






  • 2




    And its friend the closed graph theoerem.
    – ncmathsadist
    Jul 2 '12 at 14:07










  • @FrancisAdams Ooh, nice, thank you! Now I have two favourite answers. Why don't you make this comment into an answer so that I can upvote it?
    – Rudy the Reindeer
    Jul 2 '12 at 14:08










  • @ MattN It looks like someone already did.
    – Francis Adams
    Jul 2 '12 at 14:23










  • @FrancisAdams Yes I saw. Btw, if you put a space in between "@" and my name I won't get pinged.
    – Rudy the Reindeer
    Jul 2 '12 at 19:14



















28














Let $I=[0,1]$ and $mathcal{C}(I)= { f : I to mathbb{R} text{continuous} }$ with the topology of uniform convergence. Then the set of nowhere differentiable functions over $I$ is dense in $mathcal{C}(I)$.



The same thing holds in $mathcal{C}(I)$ for the set of nowhere locally monotonic functions.






share|cite|improve this answer



















  • 1




    It's in fact possible to prove the slightly stronger result that the set of functions in $mathcal C(I)$ that are differentiable at a single point is meagre (or "of the first category") in $mathcal C(I)$
    – kahen
    Aug 9 '12 at 12:10



















27














There exist $2pi$-periodic continuous functions whose Fourier series diverge on an uncountable set.






share|cite|improve this answer



















  • 13




    I'm surprised that you have only one favourite! :-)
    – Willie Wong
    Jul 3 '12 at 9:24



















24














$overline{mathbb Q_p}$ is not complete with respect to the $p$-adic absolute value.
This follows from the fact that $overline{ mathbb{Q}_p}$ has countably infinite dimension over $mathbb{Q}_p$ which can be proved using Krasner's lemma.






share|cite|improve this answer































    22














    It can show that an infinite dimensional Banach space has no countable basis.



    Firstly, assume that the Banach space $V$ has countable basis ${x_1,x_2,dots}$, and let $V_n=operatorname{span}{x_1,x_2,dots,x_n}$. It is not difficult to show that $V_n$ are closed and nowhere dense but by Baire category, $cup V_n=V$ is impossible. As a result,$V$ must has uncountable basis.






    share|cite|improve this answer



















    • 5




      You mean a countable Hamel basis, and that is exactly what Matt wrote in the question about polynomials.
      – Asaf Karagila
      Jul 2 '12 at 15:37










    • right,they are essentially the same.Thanks for pointing out this.
      – Ben
      Jul 2 '12 at 15:41



















    21














    The rationals are not completely metrizable.



    Proof: Since the rationals have no isolated points, $mathbb Qsetminus{q}$ is dense and open for every $q$, but $bigcap_{qinmathbb Q}mathbb Qsetminus{q}$ is an intersection of countably many open dense sets which is empty.



    One nice corollary from this (see Nate Eldredge's comment below) is that the rationals are not a $G_delta$ set of real numbers. Thus we have an example of an $F_sigma$ which is not $G_delta$.






    share|cite|improve this answer























    • This one?
      – Rudy the Reindeer
      Jul 2 '12 at 13:30






    • 3




      What's long about "since it's the union of its countably many points and no point is open"? :)
      – t.b.
      Jul 2 '12 at 13:44






    • 1




      @t.b. I am also attending a lecture and was trying to prove something else in my head (and yes, the generic topological space is Hausdorff). Doing that whilst typing an answer is highly nontrivial!
      – Asaf Karagila
      Jul 2 '12 at 14:12






    • 2




      @AsafKaragila You could still add the proof...
      – Rudy the Reindeer
      Jul 2 '12 at 19:39






    • 4




      You don't need Lavrentyev's theorem here. If $mathbb{Q}$ were $G_delta$ in $mathbb{R}$ it would be a dense $G_delta$, in particular comeager. But $mathbb{Q}$ is also meager. This would imply that $mathbb{R}$ is meager, which by BCT it is not.
      – Nate Eldredge
      Jul 3 '12 at 1:05



















    17














    Here is another cool one:




    Theorem. There exists a continuous function $f:[0,1] to mathbb{R}$ that is not monotone on any interval of positive length.







    share|cite|improve this answer































      17














      There is a partial differential equation with no solutions. Specifically, a first-order PDE on $mathbb{R} times mathbb{C}$ with smooth coefficients, of the form
      $$frac{partial u }{partial bar{z}} - i z frac{partial u}{partial t} = F(t,z).$$



      See Lewy's example. I don't have the proof in front of me, but as I recall it goes by showing that the collection of $F$ for which a solution exists is meager in some appropriate space.






      share|cite|improve this answer































        15














        I'm partial to the Principle of Dependent Choices, myself (which is equivalent to the BCT).






        share|cite|improve this answer



















        • 1




          Drats. I was gonna post that one when I got home, but I see I got here seven minutes too late! Damn buses! :-)
          – Asaf Karagila
          Jul 2 '12 at 17:16










        • Blair's article is unique. Short and concise. This piece of mathematical work is very special for me because was the first article that I read.
          – Paulo Henrique
          Jul 3 '12 at 23:44










        • @Fëanor: I haven't read it. What is the title and where can I find it?
          – Cameron Buie
          Jul 4 '12 at 15:16






        • 2




          @CameronBuie the article is: The Baire category theorem implies the principle of dependent choices, Bull. Acad. Polon. Sci. Sér. Sci. Math. Astronom. Phys., v. 25 n. 10 (1977), pp. 933–934, by Charles Blair. But it's quite hard to find it. See more reference about it here math.stackexchange.com/questions/146910/…
          – Paulo Henrique
          Jul 4 '12 at 17:12



















        15














        I don't know if it is my favourite, but it is one of the few I know.




        THM Let $(M,d)$ be a complete metric space with no isolated points. Then $(M,d)$ is uncountable.




        PROOF Assume $M$ is countable, and let ${x_1,x_2,x_3,dots}$ be an enumeration of $M$. Since each singleton is closed, each $X_i=Xsmallsetminus {x_i}$ is open for each $i$. Moreover, each of them is dense, since each point is an accumulation point of $X$. By Baire's Theorem, $displaystylebigcap_{iinBbb N} X_i$ must be dense, hence nonempty, but it is readily seen it is empty, which is absurd. $blacktriangle$.




        COROLLARY Let $(M,d)$ be complete, $P$ a perfect subset of $M$. Then $P$ is uncountable.




        PROOF $(P,dmid_P)$ is a complete metric space with no isolated points.






        share|cite|improve this answer



















        • 1




          (1).Nitpicking: Assuming M is not the empty space. (2). Without Baire we can show by elementary means that a complete metric space $M$ has a subspace homeomorphic to the Cantor set, so, regardless of the Continuum Hypothesis, the cardinal of $M$ is at least $2^{aleph_0}.$
          – DanielWainfleet
          Dec 16 '17 at 10:45



















        13














        The open mapping theorem and closed graph theorem of functional analysis are two vital applications.






        share|cite|improve this answer































          12















          Let $(X,d)$ a compact metric space and $V$ a closed subspace of $C(X)$, vector space of continuous functions with real values, endowed with the supremum norm. We assume that each function of $V$ is Hölderian, that is, for all $fin V$, we can find $C>0$ and $0<alpha< 1$ such that
          $$forall x,yin X,quad |f(x)-f(y)|leqslant Ccdot d(x,y)^{alpha}.$$
          Then $V$ is finite dimensional.




          Define
          $$F_n:=bigcap_{x,yin [0,1]}left{fin V,|f(x)-f(y)|leqslant ncdot d(x,y)^{1/n}right}.$$
          Then $F_n$ is a closed subset of $V$. Indeed, it suffices to notice that for any fixed $x,yin[0,1]$, the set $left{fin V,|f(x)-f(y)|leqslant ncdot d(x,y)^{1/n}right}$ is a closed subset of $V$. This can be done in the following way: let $left(f_lright)_{lgeqslant 1}$ be a sequence of elements of $left{hin V,|h(x)-h(y)|leqslant ncdot d(x,y)^{1/n}right}$ which converges uniformly on $X$ to $f$. Then for each $l$, $left|f_l(x)-f_l(y)right|leqslant n d(x,y)^{1/n}$ and taking the limit $lto +infty$, we derive that $left|f (x)-f (y)right|leqslant n d(x,y)^{1/n}$.



          We assume that $d(x,y)leqslant 1$ for all $x,y$, WLOG. Let $fin V$, and $C,alpha$ associated to this $f$. Take $n$ such that $ngeqslant C$ and $frac 1n<alpha$ to get that $fin F_n$. Indeed, we have
          $$|f(x)-f(y)|leqslant Ccdot d(x,y)^alphaleqslant ncdot d(x,y)^alpha=
          nexpleft(alphalogleft(d(x,y)right)right)leqslant ncdot d(x,y)^{1/n}.$$



          By Baire's theorem, we have that $F_{n_0}$ has a non-empty interior for some $n_0$, that is, exist, $f_0in F_{n_0}$ and $r>0$ such that if $lVert f-f_0rVert_{infty}leq r$ then $fin F_n$. For $fin V$, we have
          $f_0+frac r{2(1+lVert frVert_{infty})}fin F_{n_0}$, hence
          $$|f(x)-f(y)|leqslant |f_0(x)-f_0(y)|+frac{2(1+lVert frVert_{infty})}rcdot n_0d(x,y)^{1/n_0}\
          leqslant n_0left(1++frac{2(1+lVert frVert_{infty})}rright)d(x,y)^{1/n_0}.$$
          Now, we can see that the unit ball of $V$ has a compact closure using Arzelà-Ascoli's theorem.



          An other application can be found here.






          share|cite|improve this answer































            11














            For any $1<p<infty$ $$bigcup_{q<p}ell^q not=ell^p.$$



            To see this note that $ell^q$ is meagre in $ell^p$ with respect to $lVertcdotlVert_p$ since $ell^q=bigcup_{ninmathbb{N}}{xinell^q~|~lVert xlVert^q_qleq n}$.






            share|cite|improve this answer































              9














              Here is a nice example I recently discussed in lecture.



              Recall that a function $ f:mathbb Rto mathbb R $ is Baire one iff it is the pointwise limit of a sequence of continuous functions. These functions do not need to be continuous, but Baire proved that they always have uncountably many points of continuity; in fact, the set of points of continuity of $ f $ is a comeager $ G_delta $ set.



              To prove this, one first argues that all preimages $f^{-1}(mathcal O) $ of open sets are $ F_sigma $ sets (countable unions of closed sets). Now, $ f $ is not continuous at a point $ x $ iff there is an open interval $ I $ with rational endpoints and such that $ f (x)in I $ but there is a sequence of points $ y $ converging to $ x $ with $ f (y)notin I $, that is, $$ xin f^{-1}(I)cap overline {mathbb Rsetminus f^{-1}(I)}. $$
              Note that this is a meager $ F_sigma $ set, and that therefore the set of points of discontinuity of $ f $ is a countable union of such sets, thus also a meager $ F_sigma $ set.



              It follows from this and the Baire category theorem that, in fact, the restriction of $ f $ to any nonempty closed set has a continuity point.



              (Baire went on to show that the last statement is actually equivalent to $ f $ being Baire one.)



              Since one can readily check that derivatives are Baire one, it follows that derivatives have many points of continuity.






              share|cite|improve this answer































                8














                The Casorati–Weierstrass theorem says that if $G$ is an open subset of $mathbb C$, $a$ is in $G$, and $f:Gsetminus{a}tomathbb C$ is a holomorphic function such that $limlimits_{zto a}f(z)$ does not exist in $mathbb Ccup{infty}$, then for each disk $D$ centered at $a$ (and contained in $G$), $f(Dsetminus{a})$ is a dense subset of $mathbb C$.



                Baire's theorem can be used to give a strengthening of this result with the same hypotheses (still vastly weaker than Picard, but easier to prove). There exists a dense subset $X$ of $mathbb C$ such that for each disk $D$ centered at $a$ (and contained in $G$), $f(Dsetminus{a})$ contains $X$. In particular, this implies that for each $xin X$, there is a sequence $(z_n)$ in $Gsetminus{a}$ converging to $a$ such that $f(z_n)=x$ for all $n$.



                To prove this, let $X=bigcaplimits_{n=1}^infty f{zin G:0<|z-a|<frac{1}{n}}$, note that each set in the intersection is open and dense by the open mapping theorem and the Casorati–Weierstrass theorem, apply Baire's theorem to see that $X$ is dense, and check that $X$ has the desired property.






                share|cite|improve this answer































                  5














                  Another application of the Baire category theorem is to proof the Niemytzki Plane is not normal. It is a classical method, which called category method. see here: Application of Baire category theorem in Moore plane






                  share|cite|improve this answer



















                  • 2




                    For a proof see page 10, example 2.22, here.
                    – Rudy the Reindeer
                    Jul 3 '12 at 6:43








                  • 2




                    Link (taken from comments to question): math.stackexchange.com/questions/135947
                    – sdcvvc
                    Jul 4 '12 at 12:03










                  • Interesting. This can also be answered by the "lowest-order" case of the Jones Lemma: A separable Tychonoff space with a closed discrete subspace of cardinal $2^{aleph_0}$ is not normal.
                    – DanielWainfleet
                    Dec 16 '17 at 10:53



















                  4














                  One of my favorite (albeit elementary) applications is showing that $mathbb{Q}$ is not a $G_{delta}$ set.






                  share|cite|improve this answer





























                    3














                    My favorite application of Baire is due to Daniel Fischer - thanks a lot to Daniel!!!
                    (See his Answer to Uncountable Lebesgue Null Set.)




                    There exist uncountable Lebesgue null sets over the real line.
                    (These are precisely the ones giving rise to those obscure singular continuous measures.)







                    share|cite|improve this answer































                      2














                      Baires category theorem can be very useful if you consider CW-complexes, especially if you want to prove that a space does not admit a CW-structure (in connection with considering Baire spaces).



                      For example an infinite-dimensional Hilbert space is not a CW-complex since it is a Baire space (see i.e. Let $X$ be an infinite dimensional Banach space. Prove that every Hamel basis of X is uncountable. ). Another example with a similar but extended argument can be found here https://mathoverflow.net/questions/152802/all-mapping-space-between-cw-complexes-is-a-cw-complex, it is about the cellular mapping space $Map(X,Y)$ for $X$ and $Y$ finite CW-complexes.
                      And there are of course many other of such examples.






                      share|cite|improve this answer































                        2














                        Another less known but important application of the Baire category theorem is in the proof of the Vitali-Hahn-Saks theorem. The proof can be found in these posts
                        finite measure case and infinite measure case






                        share|cite|improve this answer































                          0















                          Let $f:mathbb R^+to mathbb R$ be continuous. Suppose that for all $x>0,$ $lim_{ntoinfty} f(nx)=0$. Then $lim_{xto infty} f(x)=0$.




                          Proof: Fixing $epsilon>0$, let $K_n={x:|f(mx)|leepsilontext{ for all }mge n$}. Then the $K_n$ are closed and their union is $mathbb R^+$, so BCT implies that some $K_n$ contains an interval, $[a,b]$. This implies that $|x|<epsilon$ for all $f(x)$ in the set $$[na,nb]cup [(n+1)a,(n+1)n]cup [(n+2)a,(n+2)b]cupdots.$$ You can show this union of intervals contains an interval of the form $[M,infty)$ for some $M$, so $|f(x)|<epsilon$ for large enough $x$, completing the proof.






                          share|cite|improve this answer































                            0














                            $newcommand{wh}{widehat}$
                            $newcommand{set}[1]{{#1}}$
                            $newcommand{vp}{varphi}$




                            Theorem 1.
                            Let $M$ and $N$ be smooth manifolds and $F:Mto N$ be a surjective smooth map of constant rank.
                            Then $F$ is a smooth submersion.




                            Proof.
                            For each point $pin M$, we can find charts $(U_p,vp_p)$ and $(V_p,psi_p)$ containing $p$ and $f(p)$ respectively such that $overline{f(U_p)}subseteq V_p$ (here, just to be clear, the closure is taken in $N$).
                            In the light of the rank theorem, we can further assume without loss of generality that the following holds
                            $$
                            psi_pcirc fcirc vp_p^{-1}(x_1,ldots,x_k,x_{k+1},ldots,x_m)=(x_1,ldots,x_k,0,ldots,0),quad forall (x_1,ldots,x_m)in wh U_p
                            $$

                            We will denote $psi_pcirc fcirc vp_p^{-1}$ as $hat f_p$.
                            Note that $hat f_p(wh U_p)$ is nowhere dense in $wh V_p$.
                            Since $psi_p^{-1}:wh V_pto V_p$ is a homeomorphism, we conclude that $psi_p^{-1}(hat f_p(wh U_p))= f_p(U_p)$ is nowhere dense in $V_p$.
                            Since $overline{f_p(U_p)}subseteq V_p$, we infer that $f_p(U_p)$ is in fact nowhere dense in $N$.
                            Now since $M$ is second countable, we can find a countable subset $C$ of $M$ such that $set{U_p}_{pin C}$ covers $M$.
                            By surjectivity of $f$, we infer that $set{f_p(U_p)}_{pin C}$ covers $N$.
                            But this means that $N$ is a countable union of nowhere dense subsets.
                            Since $N$ is locally compact Hausdorff, this contradicts the fact that $N$ is a Baire space and we are done.
                            $blacksquare$






                            share|cite|improve this answer































                              -4














                              I found one beautiful application of Baire Category Theorem which is the following:



                              Let $mathcal H$ be a separable Hilbert Space with countable orthonormal basis ${u_{k}}_{k=1}^{infty}$. Fix $nin mathbb N$ consider $mathrm{Span}{u_{1},u_{2},...,u_{n}}$ then the following sets are dense in $mathcal H$.



                              $A_{i,j}:={uin mathcal H: (u,u_{i})neq (u,u_{j})}$ where $1leq i,j leq n$ and $ineq j$.



                              Proof Hints:(a) Any proper closed vector subspace of an Hilbert Space is nowhere dense.
                              (b)A closed set is nowhere dense $Leftrightarrow$ its complement is everywhere dense.






                              share|cite|improve this answer























                              • You mean to say that any closed proper subspace is nowhere dense.
                                – Asaf Karagila
                                Jul 4 '12 at 7:51










                              • @ Asaf Karagila: Yes I edited
                                – users31526
                                Jul 4 '12 at 7:56






                              • 6




                                I don't understand. What you denote by $A_{i,j}$ is a finite intersection of open and dense sets: $$A_{i,j} = bigcap_{1 leq i lt j leq n} mathcal{H} smallsetminus (u_i - u_j)^perp$$ which is obviously open and dense in $mathcal{H}$ (no need for Baire here). And: what is the Span doing here? Are you intending to take a further countable intersection over $n$? Could you please clarify?
                                – t.b.
                                Jul 4 '12 at 11:24












                              protected by user99914 Dec 17 '17 at 21:17



                              Thank you for your interest in this question.
                              Because it has attracted low-quality or spam answers that had to be removed, posting an answer now requires 10 reputation on this site (the association bonus does not count).



                              Would you like to answer one of these unanswered questions instead?














                              24 Answers
                              24






                              active

                              oldest

                              votes








                              24 Answers
                              24






                              active

                              oldest

                              votes









                              active

                              oldest

                              votes






                              active

                              oldest

                              votes









                              50














                              If $P$ is an infinitely differentiable function such that for each $x$, there is an $n$ with $P^{(n)}(x)=0$, then $P$ is a polynomial. (Note $n$ depends on $x$.) See the discussion in Math Overflow.






                              share|cite|improve this answer



















                              • 2




                                Link, taken from comments: mathoverflow.net/questions/34059/…
                                – sdcvvc
                                Jul 4 '12 at 12:03
















                              50














                              If $P$ is an infinitely differentiable function such that for each $x$, there is an $n$ with $P^{(n)}(x)=0$, then $P$ is a polynomial. (Note $n$ depends on $x$.) See the discussion in Math Overflow.






                              share|cite|improve this answer



















                              • 2




                                Link, taken from comments: mathoverflow.net/questions/34059/…
                                – sdcvvc
                                Jul 4 '12 at 12:03














                              50












                              50








                              50






                              If $P$ is an infinitely differentiable function such that for each $x$, there is an $n$ with $P^{(n)}(x)=0$, then $P$ is a polynomial. (Note $n$ depends on $x$.) See the discussion in Math Overflow.






                              share|cite|improve this answer














                              If $P$ is an infinitely differentiable function such that for each $x$, there is an $n$ with $P^{(n)}(x)=0$, then $P$ is a polynomial. (Note $n$ depends on $x$.) See the discussion in Math Overflow.







                              share|cite|improve this answer














                              share|cite|improve this answer



                              share|cite|improve this answer








                              edited Dec 16 '17 at 10:28


























                              community wiki





                              4 revs, 2 users 50%
                              David Mitra









                              • 2




                                Link, taken from comments: mathoverflow.net/questions/34059/…
                                – sdcvvc
                                Jul 4 '12 at 12:03














                              • 2




                                Link, taken from comments: mathoverflow.net/questions/34059/…
                                – sdcvvc
                                Jul 4 '12 at 12:03








                              2




                              2




                              Link, taken from comments: mathoverflow.net/questions/34059/…
                              – sdcvvc
                              Jul 4 '12 at 12:03




                              Link, taken from comments: mathoverflow.net/questions/34059/…
                              – sdcvvc
                              Jul 4 '12 at 12:03











                              39














                              The uniform boundedness principle of Functional Analysis is a very important application of the Baire Category Theorem.



                              Added: (t.b.) See also Sokal's A really simple elementary proof of the uniform boundedness theorem for a proof without Baire.






                              share|cite|improve this answer



















                              • 3




                                The other one of the big theorems from beginning functional analysis that uses the Baire Category Theorem is the open mapping theorem
                                – Francis Adams
                                Jul 2 '12 at 14:05






                              • 2




                                And its friend the closed graph theoerem.
                                – ncmathsadist
                                Jul 2 '12 at 14:07










                              • @FrancisAdams Ooh, nice, thank you! Now I have two favourite answers. Why don't you make this comment into an answer so that I can upvote it?
                                – Rudy the Reindeer
                                Jul 2 '12 at 14:08










                              • @ MattN It looks like someone already did.
                                – Francis Adams
                                Jul 2 '12 at 14:23










                              • @FrancisAdams Yes I saw. Btw, if you put a space in between "@" and my name I won't get pinged.
                                – Rudy the Reindeer
                                Jul 2 '12 at 19:14
















                              39














                              The uniform boundedness principle of Functional Analysis is a very important application of the Baire Category Theorem.



                              Added: (t.b.) See also Sokal's A really simple elementary proof of the uniform boundedness theorem for a proof without Baire.






                              share|cite|improve this answer



















                              • 3




                                The other one of the big theorems from beginning functional analysis that uses the Baire Category Theorem is the open mapping theorem
                                – Francis Adams
                                Jul 2 '12 at 14:05






                              • 2




                                And its friend the closed graph theoerem.
                                – ncmathsadist
                                Jul 2 '12 at 14:07










                              • @FrancisAdams Ooh, nice, thank you! Now I have two favourite answers. Why don't you make this comment into an answer so that I can upvote it?
                                – Rudy the Reindeer
                                Jul 2 '12 at 14:08










                              • @ MattN It looks like someone already did.
                                – Francis Adams
                                Jul 2 '12 at 14:23










                              • @FrancisAdams Yes I saw. Btw, if you put a space in between "@" and my name I won't get pinged.
                                – Rudy the Reindeer
                                Jul 2 '12 at 19:14














                              39












                              39








                              39






                              The uniform boundedness principle of Functional Analysis is a very important application of the Baire Category Theorem.



                              Added: (t.b.) See also Sokal's A really simple elementary proof of the uniform boundedness theorem for a proof without Baire.






                              share|cite|improve this answer














                              The uniform boundedness principle of Functional Analysis is a very important application of the Baire Category Theorem.



                              Added: (t.b.) See also Sokal's A really simple elementary proof of the uniform boundedness theorem for a proof without Baire.







                              share|cite|improve this answer














                              share|cite|improve this answer



                              share|cite|improve this answer








                              edited Aug 9 '12 at 12:03


























                              community wiki





                              2 revs, 2 users 67%
                              ncmathsadist










                              • 3




                                The other one of the big theorems from beginning functional analysis that uses the Baire Category Theorem is the open mapping theorem
                                – Francis Adams
                                Jul 2 '12 at 14:05






                              • 2




                                And its friend the closed graph theoerem.
                                – ncmathsadist
                                Jul 2 '12 at 14:07










                              • @FrancisAdams Ooh, nice, thank you! Now I have two favourite answers. Why don't you make this comment into an answer so that I can upvote it?
                                – Rudy the Reindeer
                                Jul 2 '12 at 14:08










                              • @ MattN It looks like someone already did.
                                – Francis Adams
                                Jul 2 '12 at 14:23










                              • @FrancisAdams Yes I saw. Btw, if you put a space in between "@" and my name I won't get pinged.
                                – Rudy the Reindeer
                                Jul 2 '12 at 19:14














                              • 3




                                The other one of the big theorems from beginning functional analysis that uses the Baire Category Theorem is the open mapping theorem
                                – Francis Adams
                                Jul 2 '12 at 14:05






                              • 2




                                And its friend the closed graph theoerem.
                                – ncmathsadist
                                Jul 2 '12 at 14:07










                              • @FrancisAdams Ooh, nice, thank you! Now I have two favourite answers. Why don't you make this comment into an answer so that I can upvote it?
                                – Rudy the Reindeer
                                Jul 2 '12 at 14:08










                              • @ MattN It looks like someone already did.
                                – Francis Adams
                                Jul 2 '12 at 14:23










                              • @FrancisAdams Yes I saw. Btw, if you put a space in between "@" and my name I won't get pinged.
                                – Rudy the Reindeer
                                Jul 2 '12 at 19:14








                              3




                              3




                              The other one of the big theorems from beginning functional analysis that uses the Baire Category Theorem is the open mapping theorem
                              – Francis Adams
                              Jul 2 '12 at 14:05




                              The other one of the big theorems from beginning functional analysis that uses the Baire Category Theorem is the open mapping theorem
                              – Francis Adams
                              Jul 2 '12 at 14:05




                              2




                              2




                              And its friend the closed graph theoerem.
                              – ncmathsadist
                              Jul 2 '12 at 14:07




                              And its friend the closed graph theoerem.
                              – ncmathsadist
                              Jul 2 '12 at 14:07












                              @FrancisAdams Ooh, nice, thank you! Now I have two favourite answers. Why don't you make this comment into an answer so that I can upvote it?
                              – Rudy the Reindeer
                              Jul 2 '12 at 14:08




                              @FrancisAdams Ooh, nice, thank you! Now I have two favourite answers. Why don't you make this comment into an answer so that I can upvote it?
                              – Rudy the Reindeer
                              Jul 2 '12 at 14:08












                              @ MattN It looks like someone already did.
                              – Francis Adams
                              Jul 2 '12 at 14:23




                              @ MattN It looks like someone already did.
                              – Francis Adams
                              Jul 2 '12 at 14:23












                              @FrancisAdams Yes I saw. Btw, if you put a space in between "@" and my name I won't get pinged.
                              – Rudy the Reindeer
                              Jul 2 '12 at 19:14




                              @FrancisAdams Yes I saw. Btw, if you put a space in between "@" and my name I won't get pinged.
                              – Rudy the Reindeer
                              Jul 2 '12 at 19:14











                              28














                              Let $I=[0,1]$ and $mathcal{C}(I)= { f : I to mathbb{R} text{continuous} }$ with the topology of uniform convergence. Then the set of nowhere differentiable functions over $I$ is dense in $mathcal{C}(I)$.



                              The same thing holds in $mathcal{C}(I)$ for the set of nowhere locally monotonic functions.






                              share|cite|improve this answer



















                              • 1




                                It's in fact possible to prove the slightly stronger result that the set of functions in $mathcal C(I)$ that are differentiable at a single point is meagre (or "of the first category") in $mathcal C(I)$
                                – kahen
                                Aug 9 '12 at 12:10
















                              28














                              Let $I=[0,1]$ and $mathcal{C}(I)= { f : I to mathbb{R} text{continuous} }$ with the topology of uniform convergence. Then the set of nowhere differentiable functions over $I$ is dense in $mathcal{C}(I)$.



                              The same thing holds in $mathcal{C}(I)$ for the set of nowhere locally monotonic functions.






                              share|cite|improve this answer



















                              • 1




                                It's in fact possible to prove the slightly stronger result that the set of functions in $mathcal C(I)$ that are differentiable at a single point is meagre (or "of the first category") in $mathcal C(I)$
                                – kahen
                                Aug 9 '12 at 12:10














                              28












                              28








                              28






                              Let $I=[0,1]$ and $mathcal{C}(I)= { f : I to mathbb{R} text{continuous} }$ with the topology of uniform convergence. Then the set of nowhere differentiable functions over $I$ is dense in $mathcal{C}(I)$.



                              The same thing holds in $mathcal{C}(I)$ for the set of nowhere locally monotonic functions.






                              share|cite|improve this answer














                              Let $I=[0,1]$ and $mathcal{C}(I)= { f : I to mathbb{R} text{continuous} }$ with the topology of uniform convergence. Then the set of nowhere differentiable functions over $I$ is dense in $mathcal{C}(I)$.



                              The same thing holds in $mathcal{C}(I)$ for the set of nowhere locally monotonic functions.







                              share|cite|improve this answer














                              share|cite|improve this answer



                              share|cite|improve this answer








                              edited Apr 17 '13 at 14:18


























                              community wiki





                              2 revs
                              Seirios









                              • 1




                                It's in fact possible to prove the slightly stronger result that the set of functions in $mathcal C(I)$ that are differentiable at a single point is meagre (or "of the first category") in $mathcal C(I)$
                                – kahen
                                Aug 9 '12 at 12:10














                              • 1




                                It's in fact possible to prove the slightly stronger result that the set of functions in $mathcal C(I)$ that are differentiable at a single point is meagre (or "of the first category") in $mathcal C(I)$
                                – kahen
                                Aug 9 '12 at 12:10








                              1




                              1




                              It's in fact possible to prove the slightly stronger result that the set of functions in $mathcal C(I)$ that are differentiable at a single point is meagre (or "of the first category") in $mathcal C(I)$
                              – kahen
                              Aug 9 '12 at 12:10




                              It's in fact possible to prove the slightly stronger result that the set of functions in $mathcal C(I)$ that are differentiable at a single point is meagre (or "of the first category") in $mathcal C(I)$
                              – kahen
                              Aug 9 '12 at 12:10











                              27














                              There exist $2pi$-periodic continuous functions whose Fourier series diverge on an uncountable set.






                              share|cite|improve this answer



















                              • 13




                                I'm surprised that you have only one favourite! :-)
                                – Willie Wong
                                Jul 3 '12 at 9:24
















                              27














                              There exist $2pi$-periodic continuous functions whose Fourier series diverge on an uncountable set.






                              share|cite|improve this answer



















                              • 13




                                I'm surprised that you have only one favourite! :-)
                                – Willie Wong
                                Jul 3 '12 at 9:24














                              27












                              27








                              27






                              There exist $2pi$-periodic continuous functions whose Fourier series diverge on an uncountable set.






                              share|cite|improve this answer














                              There exist $2pi$-periodic continuous functions whose Fourier series diverge on an uncountable set.







                              share|cite|improve this answer














                              share|cite|improve this answer



                              share|cite|improve this answer








                              answered Jul 3 '12 at 7:28


























                              community wiki





                              Robert Israel









                              • 13




                                I'm surprised that you have only one favourite! :-)
                                – Willie Wong
                                Jul 3 '12 at 9:24














                              • 13




                                I'm surprised that you have only one favourite! :-)
                                – Willie Wong
                                Jul 3 '12 at 9:24








                              13




                              13




                              I'm surprised that you have only one favourite! :-)
                              – Willie Wong
                              Jul 3 '12 at 9:24




                              I'm surprised that you have only one favourite! :-)
                              – Willie Wong
                              Jul 3 '12 at 9:24











                              24














                              $overline{mathbb Q_p}$ is not complete with respect to the $p$-adic absolute value.
                              This follows from the fact that $overline{ mathbb{Q}_p}$ has countably infinite dimension over $mathbb{Q}_p$ which can be proved using Krasner's lemma.






                              share|cite|improve this answer




























                                24














                                $overline{mathbb Q_p}$ is not complete with respect to the $p$-adic absolute value.
                                This follows from the fact that $overline{ mathbb{Q}_p}$ has countably infinite dimension over $mathbb{Q}_p$ which can be proved using Krasner's lemma.






                                share|cite|improve this answer


























                                  24












                                  24








                                  24






                                  $overline{mathbb Q_p}$ is not complete with respect to the $p$-adic absolute value.
                                  This follows from the fact that $overline{ mathbb{Q}_p}$ has countably infinite dimension over $mathbb{Q}_p$ which can be proved using Krasner's lemma.






                                  share|cite|improve this answer














                                  $overline{mathbb Q_p}$ is not complete with respect to the $p$-adic absolute value.
                                  This follows from the fact that $overline{ mathbb{Q}_p}$ has countably infinite dimension over $mathbb{Q}_p$ which can be proved using Krasner's lemma.







                                  share|cite|improve this answer














                                  share|cite|improve this answer



                                  share|cite|improve this answer








                                  answered Jul 2 '12 at 16:14


























                                  community wiki





                                  marlu
























                                      22














                                      It can show that an infinite dimensional Banach space has no countable basis.



                                      Firstly, assume that the Banach space $V$ has countable basis ${x_1,x_2,dots}$, and let $V_n=operatorname{span}{x_1,x_2,dots,x_n}$. It is not difficult to show that $V_n$ are closed and nowhere dense but by Baire category, $cup V_n=V$ is impossible. As a result,$V$ must has uncountable basis.






                                      share|cite|improve this answer



















                                      • 5




                                        You mean a countable Hamel basis, and that is exactly what Matt wrote in the question about polynomials.
                                        – Asaf Karagila
                                        Jul 2 '12 at 15:37










                                      • right,they are essentially the same.Thanks for pointing out this.
                                        – Ben
                                        Jul 2 '12 at 15:41
















                                      22














                                      It can show that an infinite dimensional Banach space has no countable basis.



                                      Firstly, assume that the Banach space $V$ has countable basis ${x_1,x_2,dots}$, and let $V_n=operatorname{span}{x_1,x_2,dots,x_n}$. It is not difficult to show that $V_n$ are closed and nowhere dense but by Baire category, $cup V_n=V$ is impossible. As a result,$V$ must has uncountable basis.






                                      share|cite|improve this answer



















                                      • 5




                                        You mean a countable Hamel basis, and that is exactly what Matt wrote in the question about polynomials.
                                        – Asaf Karagila
                                        Jul 2 '12 at 15:37










                                      • right,they are essentially the same.Thanks for pointing out this.
                                        – Ben
                                        Jul 2 '12 at 15:41














                                      22












                                      22








                                      22






                                      It can show that an infinite dimensional Banach space has no countable basis.



                                      Firstly, assume that the Banach space $V$ has countable basis ${x_1,x_2,dots}$, and let $V_n=operatorname{span}{x_1,x_2,dots,x_n}$. It is not difficult to show that $V_n$ are closed and nowhere dense but by Baire category, $cup V_n=V$ is impossible. As a result,$V$ must has uncountable basis.






                                      share|cite|improve this answer














                                      It can show that an infinite dimensional Banach space has no countable basis.



                                      Firstly, assume that the Banach space $V$ has countable basis ${x_1,x_2,dots}$, and let $V_n=operatorname{span}{x_1,x_2,dots,x_n}$. It is not difficult to show that $V_n$ are closed and nowhere dense but by Baire category, $cup V_n=V$ is impossible. As a result,$V$ must has uncountable basis.







                                      share|cite|improve this answer














                                      share|cite|improve this answer



                                      share|cite|improve this answer








                                      edited Jul 2 '12 at 15:27


























                                      community wiki





                                      3 revs, 2 users 50%
                                      Ben










                                      • 5




                                        You mean a countable Hamel basis, and that is exactly what Matt wrote in the question about polynomials.
                                        – Asaf Karagila
                                        Jul 2 '12 at 15:37










                                      • right,they are essentially the same.Thanks for pointing out this.
                                        – Ben
                                        Jul 2 '12 at 15:41














                                      • 5




                                        You mean a countable Hamel basis, and that is exactly what Matt wrote in the question about polynomials.
                                        – Asaf Karagila
                                        Jul 2 '12 at 15:37










                                      • right,they are essentially the same.Thanks for pointing out this.
                                        – Ben
                                        Jul 2 '12 at 15:41








                                      5




                                      5




                                      You mean a countable Hamel basis, and that is exactly what Matt wrote in the question about polynomials.
                                      – Asaf Karagila
                                      Jul 2 '12 at 15:37




                                      You mean a countable Hamel basis, and that is exactly what Matt wrote in the question about polynomials.
                                      – Asaf Karagila
                                      Jul 2 '12 at 15:37












                                      right,they are essentially the same.Thanks for pointing out this.
                                      – Ben
                                      Jul 2 '12 at 15:41




                                      right,they are essentially the same.Thanks for pointing out this.
                                      – Ben
                                      Jul 2 '12 at 15:41











                                      21














                                      The rationals are not completely metrizable.



                                      Proof: Since the rationals have no isolated points, $mathbb Qsetminus{q}$ is dense and open for every $q$, but $bigcap_{qinmathbb Q}mathbb Qsetminus{q}$ is an intersection of countably many open dense sets which is empty.



                                      One nice corollary from this (see Nate Eldredge's comment below) is that the rationals are not a $G_delta$ set of real numbers. Thus we have an example of an $F_sigma$ which is not $G_delta$.






                                      share|cite|improve this answer























                                      • This one?
                                        – Rudy the Reindeer
                                        Jul 2 '12 at 13:30






                                      • 3




                                        What's long about "since it's the union of its countably many points and no point is open"? :)
                                        – t.b.
                                        Jul 2 '12 at 13:44






                                      • 1




                                        @t.b. I am also attending a lecture and was trying to prove something else in my head (and yes, the generic topological space is Hausdorff). Doing that whilst typing an answer is highly nontrivial!
                                        – Asaf Karagila
                                        Jul 2 '12 at 14:12






                                      • 2




                                        @AsafKaragila You could still add the proof...
                                        – Rudy the Reindeer
                                        Jul 2 '12 at 19:39






                                      • 4




                                        You don't need Lavrentyev's theorem here. If $mathbb{Q}$ were $G_delta$ in $mathbb{R}$ it would be a dense $G_delta$, in particular comeager. But $mathbb{Q}$ is also meager. This would imply that $mathbb{R}$ is meager, which by BCT it is not.
                                        – Nate Eldredge
                                        Jul 3 '12 at 1:05
















                                      21














                                      The rationals are not completely metrizable.



                                      Proof: Since the rationals have no isolated points, $mathbb Qsetminus{q}$ is dense and open for every $q$, but $bigcap_{qinmathbb Q}mathbb Qsetminus{q}$ is an intersection of countably many open dense sets which is empty.



                                      One nice corollary from this (see Nate Eldredge's comment below) is that the rationals are not a $G_delta$ set of real numbers. Thus we have an example of an $F_sigma$ which is not $G_delta$.






                                      share|cite|improve this answer























                                      • This one?
                                        – Rudy the Reindeer
                                        Jul 2 '12 at 13:30






                                      • 3




                                        What's long about "since it's the union of its countably many points and no point is open"? :)
                                        – t.b.
                                        Jul 2 '12 at 13:44






                                      • 1




                                        @t.b. I am also attending a lecture and was trying to prove something else in my head (and yes, the generic topological space is Hausdorff). Doing that whilst typing an answer is highly nontrivial!
                                        – Asaf Karagila
                                        Jul 2 '12 at 14:12






                                      • 2




                                        @AsafKaragila You could still add the proof...
                                        – Rudy the Reindeer
                                        Jul 2 '12 at 19:39






                                      • 4




                                        You don't need Lavrentyev's theorem here. If $mathbb{Q}$ were $G_delta$ in $mathbb{R}$ it would be a dense $G_delta$, in particular comeager. But $mathbb{Q}$ is also meager. This would imply that $mathbb{R}$ is meager, which by BCT it is not.
                                        – Nate Eldredge
                                        Jul 3 '12 at 1:05














                                      21












                                      21








                                      21






                                      The rationals are not completely metrizable.



                                      Proof: Since the rationals have no isolated points, $mathbb Qsetminus{q}$ is dense and open for every $q$, but $bigcap_{qinmathbb Q}mathbb Qsetminus{q}$ is an intersection of countably many open dense sets which is empty.



                                      One nice corollary from this (see Nate Eldredge's comment below) is that the rationals are not a $G_delta$ set of real numbers. Thus we have an example of an $F_sigma$ which is not $G_delta$.






                                      share|cite|improve this answer














                                      The rationals are not completely metrizable.



                                      Proof: Since the rationals have no isolated points, $mathbb Qsetminus{q}$ is dense and open for every $q$, but $bigcap_{qinmathbb Q}mathbb Qsetminus{q}$ is an intersection of countably many open dense sets which is empty.



                                      One nice corollary from this (see Nate Eldredge's comment below) is that the rationals are not a $G_delta$ set of real numbers. Thus we have an example of an $F_sigma$ which is not $G_delta$.







                                      share|cite|improve this answer














                                      share|cite|improve this answer



                                      share|cite|improve this answer








                                      edited Jul 3 '12 at 6:45


























                                      community wiki





                                      3 revs
                                      Asaf Karagila













                                      • This one?
                                        – Rudy the Reindeer
                                        Jul 2 '12 at 13:30






                                      • 3




                                        What's long about "since it's the union of its countably many points and no point is open"? :)
                                        – t.b.
                                        Jul 2 '12 at 13:44






                                      • 1




                                        @t.b. I am also attending a lecture and was trying to prove something else in my head (and yes, the generic topological space is Hausdorff). Doing that whilst typing an answer is highly nontrivial!
                                        – Asaf Karagila
                                        Jul 2 '12 at 14:12






                                      • 2




                                        @AsafKaragila You could still add the proof...
                                        – Rudy the Reindeer
                                        Jul 2 '12 at 19:39






                                      • 4




                                        You don't need Lavrentyev's theorem here. If $mathbb{Q}$ were $G_delta$ in $mathbb{R}$ it would be a dense $G_delta$, in particular comeager. But $mathbb{Q}$ is also meager. This would imply that $mathbb{R}$ is meager, which by BCT it is not.
                                        – Nate Eldredge
                                        Jul 3 '12 at 1:05


















                                      • This one?
                                        – Rudy the Reindeer
                                        Jul 2 '12 at 13:30






                                      • 3




                                        What's long about "since it's the union of its countably many points and no point is open"? :)
                                        – t.b.
                                        Jul 2 '12 at 13:44






                                      • 1




                                        @t.b. I am also attending a lecture and was trying to prove something else in my head (and yes, the generic topological space is Hausdorff). Doing that whilst typing an answer is highly nontrivial!
                                        – Asaf Karagila
                                        Jul 2 '12 at 14:12






                                      • 2




                                        @AsafKaragila You could still add the proof...
                                        – Rudy the Reindeer
                                        Jul 2 '12 at 19:39






                                      • 4




                                        You don't need Lavrentyev's theorem here. If $mathbb{Q}$ were $G_delta$ in $mathbb{R}$ it would be a dense $G_delta$, in particular comeager. But $mathbb{Q}$ is also meager. This would imply that $mathbb{R}$ is meager, which by BCT it is not.
                                        – Nate Eldredge
                                        Jul 3 '12 at 1:05
















                                      This one?
                                      – Rudy the Reindeer
                                      Jul 2 '12 at 13:30




                                      This one?
                                      – Rudy the Reindeer
                                      Jul 2 '12 at 13:30




                                      3




                                      3




                                      What's long about "since it's the union of its countably many points and no point is open"? :)
                                      – t.b.
                                      Jul 2 '12 at 13:44




                                      What's long about "since it's the union of its countably many points and no point is open"? :)
                                      – t.b.
                                      Jul 2 '12 at 13:44




                                      1




                                      1




                                      @t.b. I am also attending a lecture and was trying to prove something else in my head (and yes, the generic topological space is Hausdorff). Doing that whilst typing an answer is highly nontrivial!
                                      – Asaf Karagila
                                      Jul 2 '12 at 14:12




                                      @t.b. I am also attending a lecture and was trying to prove something else in my head (and yes, the generic topological space is Hausdorff). Doing that whilst typing an answer is highly nontrivial!
                                      – Asaf Karagila
                                      Jul 2 '12 at 14:12




                                      2




                                      2




                                      @AsafKaragila You could still add the proof...
                                      – Rudy the Reindeer
                                      Jul 2 '12 at 19:39




                                      @AsafKaragila You could still add the proof...
                                      – Rudy the Reindeer
                                      Jul 2 '12 at 19:39




                                      4




                                      4




                                      You don't need Lavrentyev's theorem here. If $mathbb{Q}$ were $G_delta$ in $mathbb{R}$ it would be a dense $G_delta$, in particular comeager. But $mathbb{Q}$ is also meager. This would imply that $mathbb{R}$ is meager, which by BCT it is not.
                                      – Nate Eldredge
                                      Jul 3 '12 at 1:05




                                      You don't need Lavrentyev's theorem here. If $mathbb{Q}$ were $G_delta$ in $mathbb{R}$ it would be a dense $G_delta$, in particular comeager. But $mathbb{Q}$ is also meager. This would imply that $mathbb{R}$ is meager, which by BCT it is not.
                                      – Nate Eldredge
                                      Jul 3 '12 at 1:05











                                      17














                                      Here is another cool one:




                                      Theorem. There exists a continuous function $f:[0,1] to mathbb{R}$ that is not monotone on any interval of positive length.







                                      share|cite|improve this answer




























                                        17














                                        Here is another cool one:




                                        Theorem. There exists a continuous function $f:[0,1] to mathbb{R}$ that is not monotone on any interval of positive length.







                                        share|cite|improve this answer


























                                          17












                                          17








                                          17






                                          Here is another cool one:




                                          Theorem. There exists a continuous function $f:[0,1] to mathbb{R}$ that is not monotone on any interval of positive length.







                                          share|cite|improve this answer














                                          Here is another cool one:




                                          Theorem. There exists a continuous function $f:[0,1] to mathbb{R}$ that is not monotone on any interval of positive length.








                                          share|cite|improve this answer














                                          share|cite|improve this answer



                                          share|cite|improve this answer








                                          answered Jul 3 '12 at 1:57


























                                          community wiki





                                          nullUser
























                                              17














                                              There is a partial differential equation with no solutions. Specifically, a first-order PDE on $mathbb{R} times mathbb{C}$ with smooth coefficients, of the form
                                              $$frac{partial u }{partial bar{z}} - i z frac{partial u}{partial t} = F(t,z).$$



                                              See Lewy's example. I don't have the proof in front of me, but as I recall it goes by showing that the collection of $F$ for which a solution exists is meager in some appropriate space.






                                              share|cite|improve this answer




























                                                17














                                                There is a partial differential equation with no solutions. Specifically, a first-order PDE on $mathbb{R} times mathbb{C}$ with smooth coefficients, of the form
                                                $$frac{partial u }{partial bar{z}} - i z frac{partial u}{partial t} = F(t,z).$$



                                                See Lewy's example. I don't have the proof in front of me, but as I recall it goes by showing that the collection of $F$ for which a solution exists is meager in some appropriate space.






                                                share|cite|improve this answer


























                                                  17












                                                  17








                                                  17






                                                  There is a partial differential equation with no solutions. Specifically, a first-order PDE on $mathbb{R} times mathbb{C}$ with smooth coefficients, of the form
                                                  $$frac{partial u }{partial bar{z}} - i z frac{partial u}{partial t} = F(t,z).$$



                                                  See Lewy's example. I don't have the proof in front of me, but as I recall it goes by showing that the collection of $F$ for which a solution exists is meager in some appropriate space.






                                                  share|cite|improve this answer














                                                  There is a partial differential equation with no solutions. Specifically, a first-order PDE on $mathbb{R} times mathbb{C}$ with smooth coefficients, of the form
                                                  $$frac{partial u }{partial bar{z}} - i z frac{partial u}{partial t} = F(t,z).$$



                                                  See Lewy's example. I don't have the proof in front of me, but as I recall it goes by showing that the collection of $F$ for which a solution exists is meager in some appropriate space.







                                                  share|cite|improve this answer














                                                  share|cite|improve this answer



                                                  share|cite|improve this answer








                                                  answered Mar 31 '13 at 15:17


























                                                  community wiki





                                                  Nate Eldredge
























                                                      15














                                                      I'm partial to the Principle of Dependent Choices, myself (which is equivalent to the BCT).






                                                      share|cite|improve this answer



















                                                      • 1




                                                        Drats. I was gonna post that one when I got home, but I see I got here seven minutes too late! Damn buses! :-)
                                                        – Asaf Karagila
                                                        Jul 2 '12 at 17:16










                                                      • Blair's article is unique. Short and concise. This piece of mathematical work is very special for me because was the first article that I read.
                                                        – Paulo Henrique
                                                        Jul 3 '12 at 23:44










                                                      • @Fëanor: I haven't read it. What is the title and where can I find it?
                                                        – Cameron Buie
                                                        Jul 4 '12 at 15:16






                                                      • 2




                                                        @CameronBuie the article is: The Baire category theorem implies the principle of dependent choices, Bull. Acad. Polon. Sci. Sér. Sci. Math. Astronom. Phys., v. 25 n. 10 (1977), pp. 933–934, by Charles Blair. But it's quite hard to find it. See more reference about it here math.stackexchange.com/questions/146910/…
                                                        – Paulo Henrique
                                                        Jul 4 '12 at 17:12
















                                                      15














                                                      I'm partial to the Principle of Dependent Choices, myself (which is equivalent to the BCT).






                                                      share|cite|improve this answer



















                                                      • 1




                                                        Drats. I was gonna post that one when I got home, but I see I got here seven minutes too late! Damn buses! :-)
                                                        – Asaf Karagila
                                                        Jul 2 '12 at 17:16










                                                      • Blair's article is unique. Short and concise. This piece of mathematical work is very special for me because was the first article that I read.
                                                        – Paulo Henrique
                                                        Jul 3 '12 at 23:44










                                                      • @Fëanor: I haven't read it. What is the title and where can I find it?
                                                        – Cameron Buie
                                                        Jul 4 '12 at 15:16






                                                      • 2




                                                        @CameronBuie the article is: The Baire category theorem implies the principle of dependent choices, Bull. Acad. Polon. Sci. Sér. Sci. Math. Astronom. Phys., v. 25 n. 10 (1977), pp. 933–934, by Charles Blair. But it's quite hard to find it. See more reference about it here math.stackexchange.com/questions/146910/…
                                                        – Paulo Henrique
                                                        Jul 4 '12 at 17:12














                                                      15












                                                      15








                                                      15






                                                      I'm partial to the Principle of Dependent Choices, myself (which is equivalent to the BCT).






                                                      share|cite|improve this answer














                                                      I'm partial to the Principle of Dependent Choices, myself (which is equivalent to the BCT).







                                                      share|cite|improve this answer














                                                      share|cite|improve this answer



                                                      share|cite|improve this answer








                                                      answered Jul 2 '12 at 17:07


























                                                      community wiki





                                                      Cameron Buie









                                                      • 1




                                                        Drats. I was gonna post that one when I got home, but I see I got here seven minutes too late! Damn buses! :-)
                                                        – Asaf Karagila
                                                        Jul 2 '12 at 17:16










                                                      • Blair's article is unique. Short and concise. This piece of mathematical work is very special for me because was the first article that I read.
                                                        – Paulo Henrique
                                                        Jul 3 '12 at 23:44










                                                      • @Fëanor: I haven't read it. What is the title and where can I find it?
                                                        – Cameron Buie
                                                        Jul 4 '12 at 15:16






                                                      • 2




                                                        @CameronBuie the article is: The Baire category theorem implies the principle of dependent choices, Bull. Acad. Polon. Sci. Sér. Sci. Math. Astronom. Phys., v. 25 n. 10 (1977), pp. 933–934, by Charles Blair. But it's quite hard to find it. See more reference about it here math.stackexchange.com/questions/146910/…
                                                        – Paulo Henrique
                                                        Jul 4 '12 at 17:12














                                                      • 1




                                                        Drats. I was gonna post that one when I got home, but I see I got here seven minutes too late! Damn buses! :-)
                                                        – Asaf Karagila
                                                        Jul 2 '12 at 17:16










                                                      • Blair's article is unique. Short and concise. This piece of mathematical work is very special for me because was the first article that I read.
                                                        – Paulo Henrique
                                                        Jul 3 '12 at 23:44










                                                      • @Fëanor: I haven't read it. What is the title and where can I find it?
                                                        – Cameron Buie
                                                        Jul 4 '12 at 15:16






                                                      • 2




                                                        @CameronBuie the article is: The Baire category theorem implies the principle of dependent choices, Bull. Acad. Polon. Sci. Sér. Sci. Math. Astronom. Phys., v. 25 n. 10 (1977), pp. 933–934, by Charles Blair. But it's quite hard to find it. See more reference about it here math.stackexchange.com/questions/146910/…
                                                        – Paulo Henrique
                                                        Jul 4 '12 at 17:12








                                                      1




                                                      1




                                                      Drats. I was gonna post that one when I got home, but I see I got here seven minutes too late! Damn buses! :-)
                                                      – Asaf Karagila
                                                      Jul 2 '12 at 17:16




                                                      Drats. I was gonna post that one when I got home, but I see I got here seven minutes too late! Damn buses! :-)
                                                      – Asaf Karagila
                                                      Jul 2 '12 at 17:16












                                                      Blair's article is unique. Short and concise. This piece of mathematical work is very special for me because was the first article that I read.
                                                      – Paulo Henrique
                                                      Jul 3 '12 at 23:44




                                                      Blair's article is unique. Short and concise. This piece of mathematical work is very special for me because was the first article that I read.
                                                      – Paulo Henrique
                                                      Jul 3 '12 at 23:44












                                                      @Fëanor: I haven't read it. What is the title and where can I find it?
                                                      – Cameron Buie
                                                      Jul 4 '12 at 15:16




                                                      @Fëanor: I haven't read it. What is the title and where can I find it?
                                                      – Cameron Buie
                                                      Jul 4 '12 at 15:16




                                                      2




                                                      2




                                                      @CameronBuie the article is: The Baire category theorem implies the principle of dependent choices, Bull. Acad. Polon. Sci. Sér. Sci. Math. Astronom. Phys., v. 25 n. 10 (1977), pp. 933–934, by Charles Blair. But it's quite hard to find it. See more reference about it here math.stackexchange.com/questions/146910/…
                                                      – Paulo Henrique
                                                      Jul 4 '12 at 17:12




                                                      @CameronBuie the article is: The Baire category theorem implies the principle of dependent choices, Bull. Acad. Polon. Sci. Sér. Sci. Math. Astronom. Phys., v. 25 n. 10 (1977), pp. 933–934, by Charles Blair. But it's quite hard to find it. See more reference about it here math.stackexchange.com/questions/146910/…
                                                      – Paulo Henrique
                                                      Jul 4 '12 at 17:12











                                                      15














                                                      I don't know if it is my favourite, but it is one of the few I know.




                                                      THM Let $(M,d)$ be a complete metric space with no isolated points. Then $(M,d)$ is uncountable.




                                                      PROOF Assume $M$ is countable, and let ${x_1,x_2,x_3,dots}$ be an enumeration of $M$. Since each singleton is closed, each $X_i=Xsmallsetminus {x_i}$ is open for each $i$. Moreover, each of them is dense, since each point is an accumulation point of $X$. By Baire's Theorem, $displaystylebigcap_{iinBbb N} X_i$ must be dense, hence nonempty, but it is readily seen it is empty, which is absurd. $blacktriangle$.




                                                      COROLLARY Let $(M,d)$ be complete, $P$ a perfect subset of $M$. Then $P$ is uncountable.




                                                      PROOF $(P,dmid_P)$ is a complete metric space with no isolated points.






                                                      share|cite|improve this answer



















                                                      • 1




                                                        (1).Nitpicking: Assuming M is not the empty space. (2). Without Baire we can show by elementary means that a complete metric space $M$ has a subspace homeomorphic to the Cantor set, so, regardless of the Continuum Hypothesis, the cardinal of $M$ is at least $2^{aleph_0}.$
                                                        – DanielWainfleet
                                                        Dec 16 '17 at 10:45
















                                                      15














                                                      I don't know if it is my favourite, but it is one of the few I know.




                                                      THM Let $(M,d)$ be a complete metric space with no isolated points. Then $(M,d)$ is uncountable.




                                                      PROOF Assume $M$ is countable, and let ${x_1,x_2,x_3,dots}$ be an enumeration of $M$. Since each singleton is closed, each $X_i=Xsmallsetminus {x_i}$ is open for each $i$. Moreover, each of them is dense, since each point is an accumulation point of $X$. By Baire's Theorem, $displaystylebigcap_{iinBbb N} X_i$ must be dense, hence nonempty, but it is readily seen it is empty, which is absurd. $blacktriangle$.




                                                      COROLLARY Let $(M,d)$ be complete, $P$ a perfect subset of $M$. Then $P$ is uncountable.




                                                      PROOF $(P,dmid_P)$ is a complete metric space with no isolated points.






                                                      share|cite|improve this answer



















                                                      • 1




                                                        (1).Nitpicking: Assuming M is not the empty space. (2). Without Baire we can show by elementary means that a complete metric space $M$ has a subspace homeomorphic to the Cantor set, so, regardless of the Continuum Hypothesis, the cardinal of $M$ is at least $2^{aleph_0}.$
                                                        – DanielWainfleet
                                                        Dec 16 '17 at 10:45














                                                      15












                                                      15








                                                      15






                                                      I don't know if it is my favourite, but it is one of the few I know.




                                                      THM Let $(M,d)$ be a complete metric space with no isolated points. Then $(M,d)$ is uncountable.




                                                      PROOF Assume $M$ is countable, and let ${x_1,x_2,x_3,dots}$ be an enumeration of $M$. Since each singleton is closed, each $X_i=Xsmallsetminus {x_i}$ is open for each $i$. Moreover, each of them is dense, since each point is an accumulation point of $X$. By Baire's Theorem, $displaystylebigcap_{iinBbb N} X_i$ must be dense, hence nonempty, but it is readily seen it is empty, which is absurd. $blacktriangle$.




                                                      COROLLARY Let $(M,d)$ be complete, $P$ a perfect subset of $M$. Then $P$ is uncountable.




                                                      PROOF $(P,dmid_P)$ is a complete metric space with no isolated points.






                                                      share|cite|improve this answer














                                                      I don't know if it is my favourite, but it is one of the few I know.




                                                      THM Let $(M,d)$ be a complete metric space with no isolated points. Then $(M,d)$ is uncountable.




                                                      PROOF Assume $M$ is countable, and let ${x_1,x_2,x_3,dots}$ be an enumeration of $M$. Since each singleton is closed, each $X_i=Xsmallsetminus {x_i}$ is open for each $i$. Moreover, each of them is dense, since each point is an accumulation point of $X$. By Baire's Theorem, $displaystylebigcap_{iinBbb N} X_i$ must be dense, hence nonempty, but it is readily seen it is empty, which is absurd. $blacktriangle$.




                                                      COROLLARY Let $(M,d)$ be complete, $P$ a perfect subset of $M$. Then $P$ is uncountable.




                                                      PROOF $(P,dmid_P)$ is a complete metric space with no isolated points.







                                                      share|cite|improve this answer














                                                      share|cite|improve this answer



                                                      share|cite|improve this answer








                                                      answered Jun 7 '13 at 21:19


























                                                      community wiki





                                                      Pedro Tamaroff









                                                      • 1




                                                        (1).Nitpicking: Assuming M is not the empty space. (2). Without Baire we can show by elementary means that a complete metric space $M$ has a subspace homeomorphic to the Cantor set, so, regardless of the Continuum Hypothesis, the cardinal of $M$ is at least $2^{aleph_0}.$
                                                        – DanielWainfleet
                                                        Dec 16 '17 at 10:45














                                                      • 1




                                                        (1).Nitpicking: Assuming M is not the empty space. (2). Without Baire we can show by elementary means that a complete metric space $M$ has a subspace homeomorphic to the Cantor set, so, regardless of the Continuum Hypothesis, the cardinal of $M$ is at least $2^{aleph_0}.$
                                                        – DanielWainfleet
                                                        Dec 16 '17 at 10:45








                                                      1




                                                      1




                                                      (1).Nitpicking: Assuming M is not the empty space. (2). Without Baire we can show by elementary means that a complete metric space $M$ has a subspace homeomorphic to the Cantor set, so, regardless of the Continuum Hypothesis, the cardinal of $M$ is at least $2^{aleph_0}.$
                                                      – DanielWainfleet
                                                      Dec 16 '17 at 10:45




                                                      (1).Nitpicking: Assuming M is not the empty space. (2). Without Baire we can show by elementary means that a complete metric space $M$ has a subspace homeomorphic to the Cantor set, so, regardless of the Continuum Hypothesis, the cardinal of $M$ is at least $2^{aleph_0}.$
                                                      – DanielWainfleet
                                                      Dec 16 '17 at 10:45











                                                      13














                                                      The open mapping theorem and closed graph theorem of functional analysis are two vital applications.






                                                      share|cite|improve this answer




























                                                        13














                                                        The open mapping theorem and closed graph theorem of functional analysis are two vital applications.






                                                        share|cite|improve this answer


























                                                          13












                                                          13








                                                          13






                                                          The open mapping theorem and closed graph theorem of functional analysis are two vital applications.






                                                          share|cite|improve this answer














                                                          The open mapping theorem and closed graph theorem of functional analysis are two vital applications.







                                                          share|cite|improve this answer














                                                          share|cite|improve this answer



                                                          share|cite|improve this answer








                                                          answered Jul 2 '12 at 14:12


























                                                          community wiki





                                                          ncmathsadist
























                                                              12















                                                              Let $(X,d)$ a compact metric space and $V$ a closed subspace of $C(X)$, vector space of continuous functions with real values, endowed with the supremum norm. We assume that each function of $V$ is Hölderian, that is, for all $fin V$, we can find $C>0$ and $0<alpha< 1$ such that
                                                              $$forall x,yin X,quad |f(x)-f(y)|leqslant Ccdot d(x,y)^{alpha}.$$
                                                              Then $V$ is finite dimensional.




                                                              Define
                                                              $$F_n:=bigcap_{x,yin [0,1]}left{fin V,|f(x)-f(y)|leqslant ncdot d(x,y)^{1/n}right}.$$
                                                              Then $F_n$ is a closed subset of $V$. Indeed, it suffices to notice that for any fixed $x,yin[0,1]$, the set $left{fin V,|f(x)-f(y)|leqslant ncdot d(x,y)^{1/n}right}$ is a closed subset of $V$. This can be done in the following way: let $left(f_lright)_{lgeqslant 1}$ be a sequence of elements of $left{hin V,|h(x)-h(y)|leqslant ncdot d(x,y)^{1/n}right}$ which converges uniformly on $X$ to $f$. Then for each $l$, $left|f_l(x)-f_l(y)right|leqslant n d(x,y)^{1/n}$ and taking the limit $lto +infty$, we derive that $left|f (x)-f (y)right|leqslant n d(x,y)^{1/n}$.



                                                              We assume that $d(x,y)leqslant 1$ for all $x,y$, WLOG. Let $fin V$, and $C,alpha$ associated to this $f$. Take $n$ such that $ngeqslant C$ and $frac 1n<alpha$ to get that $fin F_n$. Indeed, we have
                                                              $$|f(x)-f(y)|leqslant Ccdot d(x,y)^alphaleqslant ncdot d(x,y)^alpha=
                                                              nexpleft(alphalogleft(d(x,y)right)right)leqslant ncdot d(x,y)^{1/n}.$$



                                                              By Baire's theorem, we have that $F_{n_0}$ has a non-empty interior for some $n_0$, that is, exist, $f_0in F_{n_0}$ and $r>0$ such that if $lVert f-f_0rVert_{infty}leq r$ then $fin F_n$. For $fin V$, we have
                                                              $f_0+frac r{2(1+lVert frVert_{infty})}fin F_{n_0}$, hence
                                                              $$|f(x)-f(y)|leqslant |f_0(x)-f_0(y)|+frac{2(1+lVert frVert_{infty})}rcdot n_0d(x,y)^{1/n_0}\
                                                              leqslant n_0left(1++frac{2(1+lVert frVert_{infty})}rright)d(x,y)^{1/n_0}.$$
                                                              Now, we can see that the unit ball of $V$ has a compact closure using Arzelà-Ascoli's theorem.



                                                              An other application can be found here.






                                                              share|cite|improve this answer




























                                                                12















                                                                Let $(X,d)$ a compact metric space and $V$ a closed subspace of $C(X)$, vector space of continuous functions with real values, endowed with the supremum norm. We assume that each function of $V$ is Hölderian, that is, for all $fin V$, we can find $C>0$ and $0<alpha< 1$ such that
                                                                $$forall x,yin X,quad |f(x)-f(y)|leqslant Ccdot d(x,y)^{alpha}.$$
                                                                Then $V$ is finite dimensional.




                                                                Define
                                                                $$F_n:=bigcap_{x,yin [0,1]}left{fin V,|f(x)-f(y)|leqslant ncdot d(x,y)^{1/n}right}.$$
                                                                Then $F_n$ is a closed subset of $V$. Indeed, it suffices to notice that for any fixed $x,yin[0,1]$, the set $left{fin V,|f(x)-f(y)|leqslant ncdot d(x,y)^{1/n}right}$ is a closed subset of $V$. This can be done in the following way: let $left(f_lright)_{lgeqslant 1}$ be a sequence of elements of $left{hin V,|h(x)-h(y)|leqslant ncdot d(x,y)^{1/n}right}$ which converges uniformly on $X$ to $f$. Then for each $l$, $left|f_l(x)-f_l(y)right|leqslant n d(x,y)^{1/n}$ and taking the limit $lto +infty$, we derive that $left|f (x)-f (y)right|leqslant n d(x,y)^{1/n}$.



                                                                We assume that $d(x,y)leqslant 1$ for all $x,y$, WLOG. Let $fin V$, and $C,alpha$ associated to this $f$. Take $n$ such that $ngeqslant C$ and $frac 1n<alpha$ to get that $fin F_n$. Indeed, we have
                                                                $$|f(x)-f(y)|leqslant Ccdot d(x,y)^alphaleqslant ncdot d(x,y)^alpha=
                                                                nexpleft(alphalogleft(d(x,y)right)right)leqslant ncdot d(x,y)^{1/n}.$$



                                                                By Baire's theorem, we have that $F_{n_0}$ has a non-empty interior for some $n_0$, that is, exist, $f_0in F_{n_0}$ and $r>0$ such that if $lVert f-f_0rVert_{infty}leq r$ then $fin F_n$. For $fin V$, we have
                                                                $f_0+frac r{2(1+lVert frVert_{infty})}fin F_{n_0}$, hence
                                                                $$|f(x)-f(y)|leqslant |f_0(x)-f_0(y)|+frac{2(1+lVert frVert_{infty})}rcdot n_0d(x,y)^{1/n_0}\
                                                                leqslant n_0left(1++frac{2(1+lVert frVert_{infty})}rright)d(x,y)^{1/n_0}.$$
                                                                Now, we can see that the unit ball of $V$ has a compact closure using Arzelà-Ascoli's theorem.



                                                                An other application can be found here.






                                                                share|cite|improve this answer


























                                                                  12












                                                                  12








                                                                  12







                                                                  Let $(X,d)$ a compact metric space and $V$ a closed subspace of $C(X)$, vector space of continuous functions with real values, endowed with the supremum norm. We assume that each function of $V$ is Hölderian, that is, for all $fin V$, we can find $C>0$ and $0<alpha< 1$ such that
                                                                  $$forall x,yin X,quad |f(x)-f(y)|leqslant Ccdot d(x,y)^{alpha}.$$
                                                                  Then $V$ is finite dimensional.




                                                                  Define
                                                                  $$F_n:=bigcap_{x,yin [0,1]}left{fin V,|f(x)-f(y)|leqslant ncdot d(x,y)^{1/n}right}.$$
                                                                  Then $F_n$ is a closed subset of $V$. Indeed, it suffices to notice that for any fixed $x,yin[0,1]$, the set $left{fin V,|f(x)-f(y)|leqslant ncdot d(x,y)^{1/n}right}$ is a closed subset of $V$. This can be done in the following way: let $left(f_lright)_{lgeqslant 1}$ be a sequence of elements of $left{hin V,|h(x)-h(y)|leqslant ncdot d(x,y)^{1/n}right}$ which converges uniformly on $X$ to $f$. Then for each $l$, $left|f_l(x)-f_l(y)right|leqslant n d(x,y)^{1/n}$ and taking the limit $lto +infty$, we derive that $left|f (x)-f (y)right|leqslant n d(x,y)^{1/n}$.



                                                                  We assume that $d(x,y)leqslant 1$ for all $x,y$, WLOG. Let $fin V$, and $C,alpha$ associated to this $f$. Take $n$ such that $ngeqslant C$ and $frac 1n<alpha$ to get that $fin F_n$. Indeed, we have
                                                                  $$|f(x)-f(y)|leqslant Ccdot d(x,y)^alphaleqslant ncdot d(x,y)^alpha=
                                                                  nexpleft(alphalogleft(d(x,y)right)right)leqslant ncdot d(x,y)^{1/n}.$$



                                                                  By Baire's theorem, we have that $F_{n_0}$ has a non-empty interior for some $n_0$, that is, exist, $f_0in F_{n_0}$ and $r>0$ such that if $lVert f-f_0rVert_{infty}leq r$ then $fin F_n$. For $fin V$, we have
                                                                  $f_0+frac r{2(1+lVert frVert_{infty})}fin F_{n_0}$, hence
                                                                  $$|f(x)-f(y)|leqslant |f_0(x)-f_0(y)|+frac{2(1+lVert frVert_{infty})}rcdot n_0d(x,y)^{1/n_0}\
                                                                  leqslant n_0left(1++frac{2(1+lVert frVert_{infty})}rright)d(x,y)^{1/n_0}.$$
                                                                  Now, we can see that the unit ball of $V$ has a compact closure using Arzelà-Ascoli's theorem.



                                                                  An other application can be found here.






                                                                  share|cite|improve this answer















                                                                  Let $(X,d)$ a compact metric space and $V$ a closed subspace of $C(X)$, vector space of continuous functions with real values, endowed with the supremum norm. We assume that each function of $V$ is Hölderian, that is, for all $fin V$, we can find $C>0$ and $0<alpha< 1$ such that
                                                                  $$forall x,yin X,quad |f(x)-f(y)|leqslant Ccdot d(x,y)^{alpha}.$$
                                                                  Then $V$ is finite dimensional.




                                                                  Define
                                                                  $$F_n:=bigcap_{x,yin [0,1]}left{fin V,|f(x)-f(y)|leqslant ncdot d(x,y)^{1/n}right}.$$
                                                                  Then $F_n$ is a closed subset of $V$. Indeed, it suffices to notice that for any fixed $x,yin[0,1]$, the set $left{fin V,|f(x)-f(y)|leqslant ncdot d(x,y)^{1/n}right}$ is a closed subset of $V$. This can be done in the following way: let $left(f_lright)_{lgeqslant 1}$ be a sequence of elements of $left{hin V,|h(x)-h(y)|leqslant ncdot d(x,y)^{1/n}right}$ which converges uniformly on $X$ to $f$. Then for each $l$, $left|f_l(x)-f_l(y)right|leqslant n d(x,y)^{1/n}$ and taking the limit $lto +infty$, we derive that $left|f (x)-f (y)right|leqslant n d(x,y)^{1/n}$.



                                                                  We assume that $d(x,y)leqslant 1$ for all $x,y$, WLOG. Let $fin V$, and $C,alpha$ associated to this $f$. Take $n$ such that $ngeqslant C$ and $frac 1n<alpha$ to get that $fin F_n$. Indeed, we have
                                                                  $$|f(x)-f(y)|leqslant Ccdot d(x,y)^alphaleqslant ncdot d(x,y)^alpha=
                                                                  nexpleft(alphalogleft(d(x,y)right)right)leqslant ncdot d(x,y)^{1/n}.$$



                                                                  By Baire's theorem, we have that $F_{n_0}$ has a non-empty interior for some $n_0$, that is, exist, $f_0in F_{n_0}$ and $r>0$ such that if $lVert f-f_0rVert_{infty}leq r$ then $fin F_n$. For $fin V$, we have
                                                                  $f_0+frac r{2(1+lVert frVert_{infty})}fin F_{n_0}$, hence
                                                                  $$|f(x)-f(y)|leqslant |f_0(x)-f_0(y)|+frac{2(1+lVert frVert_{infty})}rcdot n_0d(x,y)^{1/n_0}\
                                                                  leqslant n_0left(1++frac{2(1+lVert frVert_{infty})}rright)d(x,y)^{1/n_0}.$$
                                                                  Now, we can see that the unit ball of $V$ has a compact closure using Arzelà-Ascoli's theorem.



                                                                  An other application can be found here.







                                                                  share|cite|improve this answer














                                                                  share|cite|improve this answer



                                                                  share|cite|improve this answer








                                                                  edited Apr 13 '17 at 12:21


























                                                                  community wiki





                                                                  5 revs, 2 users 96%
                                                                  Davide Giraudo
























                                                                      11














                                                                      For any $1<p<infty$ $$bigcup_{q<p}ell^q not=ell^p.$$



                                                                      To see this note that $ell^q$ is meagre in $ell^p$ with respect to $lVertcdotlVert_p$ since $ell^q=bigcup_{ninmathbb{N}}{xinell^q~|~lVert xlVert^q_qleq n}$.






                                                                      share|cite|improve this answer




























                                                                        11














                                                                        For any $1<p<infty$ $$bigcup_{q<p}ell^q not=ell^p.$$



                                                                        To see this note that $ell^q$ is meagre in $ell^p$ with respect to $lVertcdotlVert_p$ since $ell^q=bigcup_{ninmathbb{N}}{xinell^q~|~lVert xlVert^q_qleq n}$.






                                                                        share|cite|improve this answer


























                                                                          11












                                                                          11








                                                                          11






                                                                          For any $1<p<infty$ $$bigcup_{q<p}ell^q not=ell^p.$$



                                                                          To see this note that $ell^q$ is meagre in $ell^p$ with respect to $lVertcdotlVert_p$ since $ell^q=bigcup_{ninmathbb{N}}{xinell^q~|~lVert xlVert^q_qleq n}$.






                                                                          share|cite|improve this answer














                                                                          For any $1<p<infty$ $$bigcup_{q<p}ell^q not=ell^p.$$



                                                                          To see this note that $ell^q$ is meagre in $ell^p$ with respect to $lVertcdotlVert_p$ since $ell^q=bigcup_{ninmathbb{N}}{xinell^q~|~lVert xlVert^q_qleq n}$.







                                                                          share|cite|improve this answer














                                                                          share|cite|improve this answer



                                                                          share|cite|improve this answer








                                                                          answered Jun 7 '13 at 20:37


























                                                                          community wiki





                                                                          Julian
























                                                                              9














                                                                              Here is a nice example I recently discussed in lecture.



                                                                              Recall that a function $ f:mathbb Rto mathbb R $ is Baire one iff it is the pointwise limit of a sequence of continuous functions. These functions do not need to be continuous, but Baire proved that they always have uncountably many points of continuity; in fact, the set of points of continuity of $ f $ is a comeager $ G_delta $ set.



                                                                              To prove this, one first argues that all preimages $f^{-1}(mathcal O) $ of open sets are $ F_sigma $ sets (countable unions of closed sets). Now, $ f $ is not continuous at a point $ x $ iff there is an open interval $ I $ with rational endpoints and such that $ f (x)in I $ but there is a sequence of points $ y $ converging to $ x $ with $ f (y)notin I $, that is, $$ xin f^{-1}(I)cap overline {mathbb Rsetminus f^{-1}(I)}. $$
                                                                              Note that this is a meager $ F_sigma $ set, and that therefore the set of points of discontinuity of $ f $ is a countable union of such sets, thus also a meager $ F_sigma $ set.



                                                                              It follows from this and the Baire category theorem that, in fact, the restriction of $ f $ to any nonempty closed set has a continuity point.



                                                                              (Baire went on to show that the last statement is actually equivalent to $ f $ being Baire one.)



                                                                              Since one can readily check that derivatives are Baire one, it follows that derivatives have many points of continuity.






                                                                              share|cite|improve this answer




























                                                                                9














                                                                                Here is a nice example I recently discussed in lecture.



                                                                                Recall that a function $ f:mathbb Rto mathbb R $ is Baire one iff it is the pointwise limit of a sequence of continuous functions. These functions do not need to be continuous, but Baire proved that they always have uncountably many points of continuity; in fact, the set of points of continuity of $ f $ is a comeager $ G_delta $ set.



                                                                                To prove this, one first argues that all preimages $f^{-1}(mathcal O) $ of open sets are $ F_sigma $ sets (countable unions of closed sets). Now, $ f $ is not continuous at a point $ x $ iff there is an open interval $ I $ with rational endpoints and such that $ f (x)in I $ but there is a sequence of points $ y $ converging to $ x $ with $ f (y)notin I $, that is, $$ xin f^{-1}(I)cap overline {mathbb Rsetminus f^{-1}(I)}. $$
                                                                                Note that this is a meager $ F_sigma $ set, and that therefore the set of points of discontinuity of $ f $ is a countable union of such sets, thus also a meager $ F_sigma $ set.



                                                                                It follows from this and the Baire category theorem that, in fact, the restriction of $ f $ to any nonempty closed set has a continuity point.



                                                                                (Baire went on to show that the last statement is actually equivalent to $ f $ being Baire one.)



                                                                                Since one can readily check that derivatives are Baire one, it follows that derivatives have many points of continuity.






                                                                                share|cite|improve this answer


























                                                                                  9












                                                                                  9








                                                                                  9






                                                                                  Here is a nice example I recently discussed in lecture.



                                                                                  Recall that a function $ f:mathbb Rto mathbb R $ is Baire one iff it is the pointwise limit of a sequence of continuous functions. These functions do not need to be continuous, but Baire proved that they always have uncountably many points of continuity; in fact, the set of points of continuity of $ f $ is a comeager $ G_delta $ set.



                                                                                  To prove this, one first argues that all preimages $f^{-1}(mathcal O) $ of open sets are $ F_sigma $ sets (countable unions of closed sets). Now, $ f $ is not continuous at a point $ x $ iff there is an open interval $ I $ with rational endpoints and such that $ f (x)in I $ but there is a sequence of points $ y $ converging to $ x $ with $ f (y)notin I $, that is, $$ xin f^{-1}(I)cap overline {mathbb Rsetminus f^{-1}(I)}. $$
                                                                                  Note that this is a meager $ F_sigma $ set, and that therefore the set of points of discontinuity of $ f $ is a countable union of such sets, thus also a meager $ F_sigma $ set.



                                                                                  It follows from this and the Baire category theorem that, in fact, the restriction of $ f $ to any nonempty closed set has a continuity point.



                                                                                  (Baire went on to show that the last statement is actually equivalent to $ f $ being Baire one.)



                                                                                  Since one can readily check that derivatives are Baire one, it follows that derivatives have many points of continuity.






                                                                                  share|cite|improve this answer














                                                                                  Here is a nice example I recently discussed in lecture.



                                                                                  Recall that a function $ f:mathbb Rto mathbb R $ is Baire one iff it is the pointwise limit of a sequence of continuous functions. These functions do not need to be continuous, but Baire proved that they always have uncountably many points of continuity; in fact, the set of points of continuity of $ f $ is a comeager $ G_delta $ set.



                                                                                  To prove this, one first argues that all preimages $f^{-1}(mathcal O) $ of open sets are $ F_sigma $ sets (countable unions of closed sets). Now, $ f $ is not continuous at a point $ x $ iff there is an open interval $ I $ with rational endpoints and such that $ f (x)in I $ but there is a sequence of points $ y $ converging to $ x $ with $ f (y)notin I $, that is, $$ xin f^{-1}(I)cap overline {mathbb Rsetminus f^{-1}(I)}. $$
                                                                                  Note that this is a meager $ F_sigma $ set, and that therefore the set of points of discontinuity of $ f $ is a countable union of such sets, thus also a meager $ F_sigma $ set.



                                                                                  It follows from this and the Baire category theorem that, in fact, the restriction of $ f $ to any nonempty closed set has a continuity point.



                                                                                  (Baire went on to show that the last statement is actually equivalent to $ f $ being Baire one.)



                                                                                  Since one can readily check that derivatives are Baire one, it follows that derivatives have many points of continuity.







                                                                                  share|cite|improve this answer














                                                                                  share|cite|improve this answer



                                                                                  share|cite|improve this answer








                                                                                  answered Oct 23 '14 at 16:06


























                                                                                  community wiki





                                                                                  Andrés E. Caicedo
























                                                                                      8














                                                                                      The Casorati–Weierstrass theorem says that if $G$ is an open subset of $mathbb C$, $a$ is in $G$, and $f:Gsetminus{a}tomathbb C$ is a holomorphic function such that $limlimits_{zto a}f(z)$ does not exist in $mathbb Ccup{infty}$, then for each disk $D$ centered at $a$ (and contained in $G$), $f(Dsetminus{a})$ is a dense subset of $mathbb C$.



                                                                                      Baire's theorem can be used to give a strengthening of this result with the same hypotheses (still vastly weaker than Picard, but easier to prove). There exists a dense subset $X$ of $mathbb C$ such that for each disk $D$ centered at $a$ (and contained in $G$), $f(Dsetminus{a})$ contains $X$. In particular, this implies that for each $xin X$, there is a sequence $(z_n)$ in $Gsetminus{a}$ converging to $a$ such that $f(z_n)=x$ for all $n$.



                                                                                      To prove this, let $X=bigcaplimits_{n=1}^infty f{zin G:0<|z-a|<frac{1}{n}}$, note that each set in the intersection is open and dense by the open mapping theorem and the Casorati–Weierstrass theorem, apply Baire's theorem to see that $X$ is dense, and check that $X$ has the desired property.






                                                                                      share|cite|improve this answer




























                                                                                        8














                                                                                        The Casorati–Weierstrass theorem says that if $G$ is an open subset of $mathbb C$, $a$ is in $G$, and $f:Gsetminus{a}tomathbb C$ is a holomorphic function such that $limlimits_{zto a}f(z)$ does not exist in $mathbb Ccup{infty}$, then for each disk $D$ centered at $a$ (and contained in $G$), $f(Dsetminus{a})$ is a dense subset of $mathbb C$.



                                                                                        Baire's theorem can be used to give a strengthening of this result with the same hypotheses (still vastly weaker than Picard, but easier to prove). There exists a dense subset $X$ of $mathbb C$ such that for each disk $D$ centered at $a$ (and contained in $G$), $f(Dsetminus{a})$ contains $X$. In particular, this implies that for each $xin X$, there is a sequence $(z_n)$ in $Gsetminus{a}$ converging to $a$ such that $f(z_n)=x$ for all $n$.



                                                                                        To prove this, let $X=bigcaplimits_{n=1}^infty f{zin G:0<|z-a|<frac{1}{n}}$, note that each set in the intersection is open and dense by the open mapping theorem and the Casorati–Weierstrass theorem, apply Baire's theorem to see that $X$ is dense, and check that $X$ has the desired property.






                                                                                        share|cite|improve this answer


























                                                                                          8












                                                                                          8








                                                                                          8






                                                                                          The Casorati–Weierstrass theorem says that if $G$ is an open subset of $mathbb C$, $a$ is in $G$, and $f:Gsetminus{a}tomathbb C$ is a holomorphic function such that $limlimits_{zto a}f(z)$ does not exist in $mathbb Ccup{infty}$, then for each disk $D$ centered at $a$ (and contained in $G$), $f(Dsetminus{a})$ is a dense subset of $mathbb C$.



                                                                                          Baire's theorem can be used to give a strengthening of this result with the same hypotheses (still vastly weaker than Picard, but easier to prove). There exists a dense subset $X$ of $mathbb C$ such that for each disk $D$ centered at $a$ (and contained in $G$), $f(Dsetminus{a})$ contains $X$. In particular, this implies that for each $xin X$, there is a sequence $(z_n)$ in $Gsetminus{a}$ converging to $a$ such that $f(z_n)=x$ for all $n$.



                                                                                          To prove this, let $X=bigcaplimits_{n=1}^infty f{zin G:0<|z-a|<frac{1}{n}}$, note that each set in the intersection is open and dense by the open mapping theorem and the Casorati–Weierstrass theorem, apply Baire's theorem to see that $X$ is dense, and check that $X$ has the desired property.






                                                                                          share|cite|improve this answer














                                                                                          The Casorati–Weierstrass theorem says that if $G$ is an open subset of $mathbb C$, $a$ is in $G$, and $f:Gsetminus{a}tomathbb C$ is a holomorphic function such that $limlimits_{zto a}f(z)$ does not exist in $mathbb Ccup{infty}$, then for each disk $D$ centered at $a$ (and contained in $G$), $f(Dsetminus{a})$ is a dense subset of $mathbb C$.



                                                                                          Baire's theorem can be used to give a strengthening of this result with the same hypotheses (still vastly weaker than Picard, but easier to prove). There exists a dense subset $X$ of $mathbb C$ such that for each disk $D$ centered at $a$ (and contained in $G$), $f(Dsetminus{a})$ contains $X$. In particular, this implies that for each $xin X$, there is a sequence $(z_n)$ in $Gsetminus{a}$ converging to $a$ such that $f(z_n)=x$ for all $n$.



                                                                                          To prove this, let $X=bigcaplimits_{n=1}^infty f{zin G:0<|z-a|<frac{1}{n}}$, note that each set in the intersection is open and dense by the open mapping theorem and the Casorati–Weierstrass theorem, apply Baire's theorem to see that $X$ is dense, and check that $X$ has the desired property.







                                                                                          share|cite|improve this answer














                                                                                          share|cite|improve this answer



                                                                                          share|cite|improve this answer








                                                                                          answered Jul 3 '12 at 7:04


























                                                                                          community wiki





                                                                                          Jonas Meyer
























                                                                                              5














                                                                                              Another application of the Baire category theorem is to proof the Niemytzki Plane is not normal. It is a classical method, which called category method. see here: Application of Baire category theorem in Moore plane






                                                                                              share|cite|improve this answer



















                                                                                              • 2




                                                                                                For a proof see page 10, example 2.22, here.
                                                                                                – Rudy the Reindeer
                                                                                                Jul 3 '12 at 6:43








                                                                                              • 2




                                                                                                Link (taken from comments to question): math.stackexchange.com/questions/135947
                                                                                                – sdcvvc
                                                                                                Jul 4 '12 at 12:03










                                                                                              • Interesting. This can also be answered by the "lowest-order" case of the Jones Lemma: A separable Tychonoff space with a closed discrete subspace of cardinal $2^{aleph_0}$ is not normal.
                                                                                                – DanielWainfleet
                                                                                                Dec 16 '17 at 10:53
















                                                                                              5














                                                                                              Another application of the Baire category theorem is to proof the Niemytzki Plane is not normal. It is a classical method, which called category method. see here: Application of Baire category theorem in Moore plane






                                                                                              share|cite|improve this answer



















                                                                                              • 2




                                                                                                For a proof see page 10, example 2.22, here.
                                                                                                – Rudy the Reindeer
                                                                                                Jul 3 '12 at 6:43








                                                                                              • 2




                                                                                                Link (taken from comments to question): math.stackexchange.com/questions/135947
                                                                                                – sdcvvc
                                                                                                Jul 4 '12 at 12:03










                                                                                              • Interesting. This can also be answered by the "lowest-order" case of the Jones Lemma: A separable Tychonoff space with a closed discrete subspace of cardinal $2^{aleph_0}$ is not normal.
                                                                                                – DanielWainfleet
                                                                                                Dec 16 '17 at 10:53














                                                                                              5












                                                                                              5








                                                                                              5






                                                                                              Another application of the Baire category theorem is to proof the Niemytzki Plane is not normal. It is a classical method, which called category method. see here: Application of Baire category theorem in Moore plane






                                                                                              share|cite|improve this answer














                                                                                              Another application of the Baire category theorem is to proof the Niemytzki Plane is not normal. It is a classical method, which called category method. see here: Application of Baire category theorem in Moore plane







                                                                                              share|cite|improve this answer














                                                                                              share|cite|improve this answer



                                                                                              share|cite|improve this answer








                                                                                              edited Apr 13 '17 at 12:19


























                                                                                              community wiki





                                                                                              4 revs
                                                                                              Paul









                                                                                              • 2




                                                                                                For a proof see page 10, example 2.22, here.
                                                                                                – Rudy the Reindeer
                                                                                                Jul 3 '12 at 6:43








                                                                                              • 2




                                                                                                Link (taken from comments to question): math.stackexchange.com/questions/135947
                                                                                                – sdcvvc
                                                                                                Jul 4 '12 at 12:03










                                                                                              • Interesting. This can also be answered by the "lowest-order" case of the Jones Lemma: A separable Tychonoff space with a closed discrete subspace of cardinal $2^{aleph_0}$ is not normal.
                                                                                                – DanielWainfleet
                                                                                                Dec 16 '17 at 10:53














                                                                                              • 2




                                                                                                For a proof see page 10, example 2.22, here.
                                                                                                – Rudy the Reindeer
                                                                                                Jul 3 '12 at 6:43








                                                                                              • 2




                                                                                                Link (taken from comments to question): math.stackexchange.com/questions/135947
                                                                                                – sdcvvc
                                                                                                Jul 4 '12 at 12:03










                                                                                              • Interesting. This can also be answered by the "lowest-order" case of the Jones Lemma: A separable Tychonoff space with a closed discrete subspace of cardinal $2^{aleph_0}$ is not normal.
                                                                                                – DanielWainfleet
                                                                                                Dec 16 '17 at 10:53








                                                                                              2




                                                                                              2




                                                                                              For a proof see page 10, example 2.22, here.
                                                                                              – Rudy the Reindeer
                                                                                              Jul 3 '12 at 6:43






                                                                                              For a proof see page 10, example 2.22, here.
                                                                                              – Rudy the Reindeer
                                                                                              Jul 3 '12 at 6:43






                                                                                              2




                                                                                              2




                                                                                              Link (taken from comments to question): math.stackexchange.com/questions/135947
                                                                                              – sdcvvc
                                                                                              Jul 4 '12 at 12:03




                                                                                              Link (taken from comments to question): math.stackexchange.com/questions/135947
                                                                                              – sdcvvc
                                                                                              Jul 4 '12 at 12:03












                                                                                              Interesting. This can also be answered by the "lowest-order" case of the Jones Lemma: A separable Tychonoff space with a closed discrete subspace of cardinal $2^{aleph_0}$ is not normal.
                                                                                              – DanielWainfleet
                                                                                              Dec 16 '17 at 10:53




                                                                                              Interesting. This can also be answered by the "lowest-order" case of the Jones Lemma: A separable Tychonoff space with a closed discrete subspace of cardinal $2^{aleph_0}$ is not normal.
                                                                                              – DanielWainfleet
                                                                                              Dec 16 '17 at 10:53











                                                                                              4














                                                                                              One of my favorite (albeit elementary) applications is showing that $mathbb{Q}$ is not a $G_{delta}$ set.






                                                                                              share|cite|improve this answer


























                                                                                                4














                                                                                                One of my favorite (albeit elementary) applications is showing that $mathbb{Q}$ is not a $G_{delta}$ set.






                                                                                                share|cite|improve this answer
























                                                                                                  4












                                                                                                  4








                                                                                                  4






                                                                                                  One of my favorite (albeit elementary) applications is showing that $mathbb{Q}$ is not a $G_{delta}$ set.






                                                                                                  share|cite|improve this answer












                                                                                                  One of my favorite (albeit elementary) applications is showing that $mathbb{Q}$ is not a $G_{delta}$ set.







                                                                                                  share|cite|improve this answer












                                                                                                  share|cite|improve this answer



                                                                                                  share|cite|improve this answer










                                                                                                  answered Apr 12 '13 at 19:09









                                                                                                  Euler....IS_ALIVE

                                                                                                  2,65311338




                                                                                                  2,65311338























                                                                                                      3














                                                                                                      My favorite application of Baire is due to Daniel Fischer - thanks a lot to Daniel!!!
                                                                                                      (See his Answer to Uncountable Lebesgue Null Set.)




                                                                                                      There exist uncountable Lebesgue null sets over the real line.
                                                                                                      (These are precisely the ones giving rise to those obscure singular continuous measures.)







                                                                                                      share|cite|improve this answer




























                                                                                                        3














                                                                                                        My favorite application of Baire is due to Daniel Fischer - thanks a lot to Daniel!!!
                                                                                                        (See his Answer to Uncountable Lebesgue Null Set.)




                                                                                                        There exist uncountable Lebesgue null sets over the real line.
                                                                                                        (These are precisely the ones giving rise to those obscure singular continuous measures.)







                                                                                                        share|cite|improve this answer


























                                                                                                          3












                                                                                                          3








                                                                                                          3






                                                                                                          My favorite application of Baire is due to Daniel Fischer - thanks a lot to Daniel!!!
                                                                                                          (See his Answer to Uncountable Lebesgue Null Set.)




                                                                                                          There exist uncountable Lebesgue null sets over the real line.
                                                                                                          (These are precisely the ones giving rise to those obscure singular continuous measures.)







                                                                                                          share|cite|improve this answer














                                                                                                          My favorite application of Baire is due to Daniel Fischer - thanks a lot to Daniel!!!
                                                                                                          (See his Answer to Uncountable Lebesgue Null Set.)




                                                                                                          There exist uncountable Lebesgue null sets over the real line.
                                                                                                          (These are precisely the ones giving rise to those obscure singular continuous measures.)








                                                                                                          share|cite|improve this answer














                                                                                                          share|cite|improve this answer



                                                                                                          share|cite|improve this answer








                                                                                                          edited Apr 13 '17 at 12:21


























                                                                                                          community wiki





                                                                                                          4 revs, 2 users 83%
                                                                                                          FreeziiS
























                                                                                                              2














                                                                                                              Baires category theorem can be very useful if you consider CW-complexes, especially if you want to prove that a space does not admit a CW-structure (in connection with considering Baire spaces).



                                                                                                              For example an infinite-dimensional Hilbert space is not a CW-complex since it is a Baire space (see i.e. Let $X$ be an infinite dimensional Banach space. Prove that every Hamel basis of X is uncountable. ). Another example with a similar but extended argument can be found here https://mathoverflow.net/questions/152802/all-mapping-space-between-cw-complexes-is-a-cw-complex, it is about the cellular mapping space $Map(X,Y)$ for $X$ and $Y$ finite CW-complexes.
                                                                                                              And there are of course many other of such examples.






                                                                                                              share|cite|improve this answer




























                                                                                                                2














                                                                                                                Baires category theorem can be very useful if you consider CW-complexes, especially if you want to prove that a space does not admit a CW-structure (in connection with considering Baire spaces).



                                                                                                                For example an infinite-dimensional Hilbert space is not a CW-complex since it is a Baire space (see i.e. Let $X$ be an infinite dimensional Banach space. Prove that every Hamel basis of X is uncountable. ). Another example with a similar but extended argument can be found here https://mathoverflow.net/questions/152802/all-mapping-space-between-cw-complexes-is-a-cw-complex, it is about the cellular mapping space $Map(X,Y)$ for $X$ and $Y$ finite CW-complexes.
                                                                                                                And there are of course many other of such examples.






                                                                                                                share|cite|improve this answer


























                                                                                                                  2












                                                                                                                  2








                                                                                                                  2






                                                                                                                  Baires category theorem can be very useful if you consider CW-complexes, especially if you want to prove that a space does not admit a CW-structure (in connection with considering Baire spaces).



                                                                                                                  For example an infinite-dimensional Hilbert space is not a CW-complex since it is a Baire space (see i.e. Let $X$ be an infinite dimensional Banach space. Prove that every Hamel basis of X is uncountable. ). Another example with a similar but extended argument can be found here https://mathoverflow.net/questions/152802/all-mapping-space-between-cw-complexes-is-a-cw-complex, it is about the cellular mapping space $Map(X,Y)$ for $X$ and $Y$ finite CW-complexes.
                                                                                                                  And there are of course many other of such examples.






                                                                                                                  share|cite|improve this answer














                                                                                                                  Baires category theorem can be very useful if you consider CW-complexes, especially if you want to prove that a space does not admit a CW-structure (in connection with considering Baire spaces).



                                                                                                                  For example an infinite-dimensional Hilbert space is not a CW-complex since it is a Baire space (see i.e. Let $X$ be an infinite dimensional Banach space. Prove that every Hamel basis of X is uncountable. ). Another example with a similar but extended argument can be found here https://mathoverflow.net/questions/152802/all-mapping-space-between-cw-complexes-is-a-cw-complex, it is about the cellular mapping space $Map(X,Y)$ for $X$ and $Y$ finite CW-complexes.
                                                                                                                  And there are of course many other of such examples.







                                                                                                                  share|cite|improve this answer














                                                                                                                  share|cite|improve this answer



                                                                                                                  share|cite|improve this answer








                                                                                                                  answered Apr 21 '17 at 2:29


























                                                                                                                  community wiki





                                                                                                                  toto
























                                                                                                                      2














                                                                                                                      Another less known but important application of the Baire category theorem is in the proof of the Vitali-Hahn-Saks theorem. The proof can be found in these posts
                                                                                                                      finite measure case and infinite measure case






                                                                                                                      share|cite|improve this answer




























                                                                                                                        2














                                                                                                                        Another less known but important application of the Baire category theorem is in the proof of the Vitali-Hahn-Saks theorem. The proof can be found in these posts
                                                                                                                        finite measure case and infinite measure case






                                                                                                                        share|cite|improve this answer


























                                                                                                                          2












                                                                                                                          2








                                                                                                                          2






                                                                                                                          Another less known but important application of the Baire category theorem is in the proof of the Vitali-Hahn-Saks theorem. The proof can be found in these posts
                                                                                                                          finite measure case and infinite measure case






                                                                                                                          share|cite|improve this answer














                                                                                                                          Another less known but important application of the Baire category theorem is in the proof of the Vitali-Hahn-Saks theorem. The proof can be found in these posts
                                                                                                                          finite measure case and infinite measure case







                                                                                                                          share|cite|improve this answer














                                                                                                                          share|cite|improve this answer



                                                                                                                          share|cite|improve this answer








                                                                                                                          answered Dec 17 '17 at 21:09


























                                                                                                                          community wiki





                                                                                                                          Gio67
























                                                                                                                              0















                                                                                                                              Let $f:mathbb R^+to mathbb R$ be continuous. Suppose that for all $x>0,$ $lim_{ntoinfty} f(nx)=0$. Then $lim_{xto infty} f(x)=0$.




                                                                                                                              Proof: Fixing $epsilon>0$, let $K_n={x:|f(mx)|leepsilontext{ for all }mge n$}. Then the $K_n$ are closed and their union is $mathbb R^+$, so BCT implies that some $K_n$ contains an interval, $[a,b]$. This implies that $|x|<epsilon$ for all $f(x)$ in the set $$[na,nb]cup [(n+1)a,(n+1)n]cup [(n+2)a,(n+2)b]cupdots.$$ You can show this union of intervals contains an interval of the form $[M,infty)$ for some $M$, so $|f(x)|<epsilon$ for large enough $x$, completing the proof.






                                                                                                                              share|cite|improve this answer




























                                                                                                                                0















                                                                                                                                Let $f:mathbb R^+to mathbb R$ be continuous. Suppose that for all $x>0,$ $lim_{ntoinfty} f(nx)=0$. Then $lim_{xto infty} f(x)=0$.




                                                                                                                                Proof: Fixing $epsilon>0$, let $K_n={x:|f(mx)|leepsilontext{ for all }mge n$}. Then the $K_n$ are closed and their union is $mathbb R^+$, so BCT implies that some $K_n$ contains an interval, $[a,b]$. This implies that $|x|<epsilon$ for all $f(x)$ in the set $$[na,nb]cup [(n+1)a,(n+1)n]cup [(n+2)a,(n+2)b]cupdots.$$ You can show this union of intervals contains an interval of the form $[M,infty)$ for some $M$, so $|f(x)|<epsilon$ for large enough $x$, completing the proof.






                                                                                                                                share|cite|improve this answer


























                                                                                                                                  0












                                                                                                                                  0








                                                                                                                                  0







                                                                                                                                  Let $f:mathbb R^+to mathbb R$ be continuous. Suppose that for all $x>0,$ $lim_{ntoinfty} f(nx)=0$. Then $lim_{xto infty} f(x)=0$.




                                                                                                                                  Proof: Fixing $epsilon>0$, let $K_n={x:|f(mx)|leepsilontext{ for all }mge n$}. Then the $K_n$ are closed and their union is $mathbb R^+$, so BCT implies that some $K_n$ contains an interval, $[a,b]$. This implies that $|x|<epsilon$ for all $f(x)$ in the set $$[na,nb]cup [(n+1)a,(n+1)n]cup [(n+2)a,(n+2)b]cupdots.$$ You can show this union of intervals contains an interval of the form $[M,infty)$ for some $M$, so $|f(x)|<epsilon$ for large enough $x$, completing the proof.






                                                                                                                                  share|cite|improve this answer















                                                                                                                                  Let $f:mathbb R^+to mathbb R$ be continuous. Suppose that for all $x>0,$ $lim_{ntoinfty} f(nx)=0$. Then $lim_{xto infty} f(x)=0$.




                                                                                                                                  Proof: Fixing $epsilon>0$, let $K_n={x:|f(mx)|leepsilontext{ for all }mge n$}. Then the $K_n$ are closed and their union is $mathbb R^+$, so BCT implies that some $K_n$ contains an interval, $[a,b]$. This implies that $|x|<epsilon$ for all $f(x)$ in the set $$[na,nb]cup [(n+1)a,(n+1)n]cup [(n+2)a,(n+2)b]cupdots.$$ You can show this union of intervals contains an interval of the form $[M,infty)$ for some $M$, so $|f(x)|<epsilon$ for large enough $x$, completing the proof.







                                                                                                                                  share|cite|improve this answer














                                                                                                                                  share|cite|improve this answer



                                                                                                                                  share|cite|improve this answer








                                                                                                                                  edited Dec 22 at 6:26


























                                                                                                                                  community wiki





                                                                                                                                  2 revs
                                                                                                                                  Mike Earnest
























                                                                                                                                      0














                                                                                                                                      $newcommand{wh}{widehat}$
                                                                                                                                      $newcommand{set}[1]{{#1}}$
                                                                                                                                      $newcommand{vp}{varphi}$




                                                                                                                                      Theorem 1.
                                                                                                                                      Let $M$ and $N$ be smooth manifolds and $F:Mto N$ be a surjective smooth map of constant rank.
                                                                                                                                      Then $F$ is a smooth submersion.




                                                                                                                                      Proof.
                                                                                                                                      For each point $pin M$, we can find charts $(U_p,vp_p)$ and $(V_p,psi_p)$ containing $p$ and $f(p)$ respectively such that $overline{f(U_p)}subseteq V_p$ (here, just to be clear, the closure is taken in $N$).
                                                                                                                                      In the light of the rank theorem, we can further assume without loss of generality that the following holds
                                                                                                                                      $$
                                                                                                                                      psi_pcirc fcirc vp_p^{-1}(x_1,ldots,x_k,x_{k+1},ldots,x_m)=(x_1,ldots,x_k,0,ldots,0),quad forall (x_1,ldots,x_m)in wh U_p
                                                                                                                                      $$

                                                                                                                                      We will denote $psi_pcirc fcirc vp_p^{-1}$ as $hat f_p$.
                                                                                                                                      Note that $hat f_p(wh U_p)$ is nowhere dense in $wh V_p$.
                                                                                                                                      Since $psi_p^{-1}:wh V_pto V_p$ is a homeomorphism, we conclude that $psi_p^{-1}(hat f_p(wh U_p))= f_p(U_p)$ is nowhere dense in $V_p$.
                                                                                                                                      Since $overline{f_p(U_p)}subseteq V_p$, we infer that $f_p(U_p)$ is in fact nowhere dense in $N$.
                                                                                                                                      Now since $M$ is second countable, we can find a countable subset $C$ of $M$ such that $set{U_p}_{pin C}$ covers $M$.
                                                                                                                                      By surjectivity of $f$, we infer that $set{f_p(U_p)}_{pin C}$ covers $N$.
                                                                                                                                      But this means that $N$ is a countable union of nowhere dense subsets.
                                                                                                                                      Since $N$ is locally compact Hausdorff, this contradicts the fact that $N$ is a Baire space and we are done.
                                                                                                                                      $blacksquare$






                                                                                                                                      share|cite|improve this answer




























                                                                                                                                        0














                                                                                                                                        $newcommand{wh}{widehat}$
                                                                                                                                        $newcommand{set}[1]{{#1}}$
                                                                                                                                        $newcommand{vp}{varphi}$




                                                                                                                                        Theorem 1.
                                                                                                                                        Let $M$ and $N$ be smooth manifolds and $F:Mto N$ be a surjective smooth map of constant rank.
                                                                                                                                        Then $F$ is a smooth submersion.




                                                                                                                                        Proof.
                                                                                                                                        For each point $pin M$, we can find charts $(U_p,vp_p)$ and $(V_p,psi_p)$ containing $p$ and $f(p)$ respectively such that $overline{f(U_p)}subseteq V_p$ (here, just to be clear, the closure is taken in $N$).
                                                                                                                                        In the light of the rank theorem, we can further assume without loss of generality that the following holds
                                                                                                                                        $$
                                                                                                                                        psi_pcirc fcirc vp_p^{-1}(x_1,ldots,x_k,x_{k+1},ldots,x_m)=(x_1,ldots,x_k,0,ldots,0),quad forall (x_1,ldots,x_m)in wh U_p
                                                                                                                                        $$

                                                                                                                                        We will denote $psi_pcirc fcirc vp_p^{-1}$ as $hat f_p$.
                                                                                                                                        Note that $hat f_p(wh U_p)$ is nowhere dense in $wh V_p$.
                                                                                                                                        Since $psi_p^{-1}:wh V_pto V_p$ is a homeomorphism, we conclude that $psi_p^{-1}(hat f_p(wh U_p))= f_p(U_p)$ is nowhere dense in $V_p$.
                                                                                                                                        Since $overline{f_p(U_p)}subseteq V_p$, we infer that $f_p(U_p)$ is in fact nowhere dense in $N$.
                                                                                                                                        Now since $M$ is second countable, we can find a countable subset $C$ of $M$ such that $set{U_p}_{pin C}$ covers $M$.
                                                                                                                                        By surjectivity of $f$, we infer that $set{f_p(U_p)}_{pin C}$ covers $N$.
                                                                                                                                        But this means that $N$ is a countable union of nowhere dense subsets.
                                                                                                                                        Since $N$ is locally compact Hausdorff, this contradicts the fact that $N$ is a Baire space and we are done.
                                                                                                                                        $blacksquare$






                                                                                                                                        share|cite|improve this answer


























                                                                                                                                          0












                                                                                                                                          0








                                                                                                                                          0






                                                                                                                                          $newcommand{wh}{widehat}$
                                                                                                                                          $newcommand{set}[1]{{#1}}$
                                                                                                                                          $newcommand{vp}{varphi}$




                                                                                                                                          Theorem 1.
                                                                                                                                          Let $M$ and $N$ be smooth manifolds and $F:Mto N$ be a surjective smooth map of constant rank.
                                                                                                                                          Then $F$ is a smooth submersion.




                                                                                                                                          Proof.
                                                                                                                                          For each point $pin M$, we can find charts $(U_p,vp_p)$ and $(V_p,psi_p)$ containing $p$ and $f(p)$ respectively such that $overline{f(U_p)}subseteq V_p$ (here, just to be clear, the closure is taken in $N$).
                                                                                                                                          In the light of the rank theorem, we can further assume without loss of generality that the following holds
                                                                                                                                          $$
                                                                                                                                          psi_pcirc fcirc vp_p^{-1}(x_1,ldots,x_k,x_{k+1},ldots,x_m)=(x_1,ldots,x_k,0,ldots,0),quad forall (x_1,ldots,x_m)in wh U_p
                                                                                                                                          $$

                                                                                                                                          We will denote $psi_pcirc fcirc vp_p^{-1}$ as $hat f_p$.
                                                                                                                                          Note that $hat f_p(wh U_p)$ is nowhere dense in $wh V_p$.
                                                                                                                                          Since $psi_p^{-1}:wh V_pto V_p$ is a homeomorphism, we conclude that $psi_p^{-1}(hat f_p(wh U_p))= f_p(U_p)$ is nowhere dense in $V_p$.
                                                                                                                                          Since $overline{f_p(U_p)}subseteq V_p$, we infer that $f_p(U_p)$ is in fact nowhere dense in $N$.
                                                                                                                                          Now since $M$ is second countable, we can find a countable subset $C$ of $M$ such that $set{U_p}_{pin C}$ covers $M$.
                                                                                                                                          By surjectivity of $f$, we infer that $set{f_p(U_p)}_{pin C}$ covers $N$.
                                                                                                                                          But this means that $N$ is a countable union of nowhere dense subsets.
                                                                                                                                          Since $N$ is locally compact Hausdorff, this contradicts the fact that $N$ is a Baire space and we are done.
                                                                                                                                          $blacksquare$






                                                                                                                                          share|cite|improve this answer














                                                                                                                                          $newcommand{wh}{widehat}$
                                                                                                                                          $newcommand{set}[1]{{#1}}$
                                                                                                                                          $newcommand{vp}{varphi}$




                                                                                                                                          Theorem 1.
                                                                                                                                          Let $M$ and $N$ be smooth manifolds and $F:Mto N$ be a surjective smooth map of constant rank.
                                                                                                                                          Then $F$ is a smooth submersion.




                                                                                                                                          Proof.
                                                                                                                                          For each point $pin M$, we can find charts $(U_p,vp_p)$ and $(V_p,psi_p)$ containing $p$ and $f(p)$ respectively such that $overline{f(U_p)}subseteq V_p$ (here, just to be clear, the closure is taken in $N$).
                                                                                                                                          In the light of the rank theorem, we can further assume without loss of generality that the following holds
                                                                                                                                          $$
                                                                                                                                          psi_pcirc fcirc vp_p^{-1}(x_1,ldots,x_k,x_{k+1},ldots,x_m)=(x_1,ldots,x_k,0,ldots,0),quad forall (x_1,ldots,x_m)in wh U_p
                                                                                                                                          $$

                                                                                                                                          We will denote $psi_pcirc fcirc vp_p^{-1}$ as $hat f_p$.
                                                                                                                                          Note that $hat f_p(wh U_p)$ is nowhere dense in $wh V_p$.
                                                                                                                                          Since $psi_p^{-1}:wh V_pto V_p$ is a homeomorphism, we conclude that $psi_p^{-1}(hat f_p(wh U_p))= f_p(U_p)$ is nowhere dense in $V_p$.
                                                                                                                                          Since $overline{f_p(U_p)}subseteq V_p$, we infer that $f_p(U_p)$ is in fact nowhere dense in $N$.
                                                                                                                                          Now since $M$ is second countable, we can find a countable subset $C$ of $M$ such that $set{U_p}_{pin C}$ covers $M$.
                                                                                                                                          By surjectivity of $f$, we infer that $set{f_p(U_p)}_{pin C}$ covers $N$.
                                                                                                                                          But this means that $N$ is a countable union of nowhere dense subsets.
                                                                                                                                          Since $N$ is locally compact Hausdorff, this contradicts the fact that $N$ is a Baire space and we are done.
                                                                                                                                          $blacksquare$







                                                                                                                                          share|cite|improve this answer














                                                                                                                                          share|cite|improve this answer



                                                                                                                                          share|cite|improve this answer








                                                                                                                                          answered Dec 26 at 7:31


























                                                                                                                                          community wiki





                                                                                                                                          caffeinemachine
























                                                                                                                                              -4














                                                                                                                                              I found one beautiful application of Baire Category Theorem which is the following:



                                                                                                                                              Let $mathcal H$ be a separable Hilbert Space with countable orthonormal basis ${u_{k}}_{k=1}^{infty}$. Fix $nin mathbb N$ consider $mathrm{Span}{u_{1},u_{2},...,u_{n}}$ then the following sets are dense in $mathcal H$.



                                                                                                                                              $A_{i,j}:={uin mathcal H: (u,u_{i})neq (u,u_{j})}$ where $1leq i,j leq n$ and $ineq j$.



                                                                                                                                              Proof Hints:(a) Any proper closed vector subspace of an Hilbert Space is nowhere dense.
                                                                                                                                              (b)A closed set is nowhere dense $Leftrightarrow$ its complement is everywhere dense.






                                                                                                                                              share|cite|improve this answer























                                                                                                                                              • You mean to say that any closed proper subspace is nowhere dense.
                                                                                                                                                – Asaf Karagila
                                                                                                                                                Jul 4 '12 at 7:51










                                                                                                                                              • @ Asaf Karagila: Yes I edited
                                                                                                                                                – users31526
                                                                                                                                                Jul 4 '12 at 7:56






                                                                                                                                              • 6




                                                                                                                                                I don't understand. What you denote by $A_{i,j}$ is a finite intersection of open and dense sets: $$A_{i,j} = bigcap_{1 leq i lt j leq n} mathcal{H} smallsetminus (u_i - u_j)^perp$$ which is obviously open and dense in $mathcal{H}$ (no need for Baire here). And: what is the Span doing here? Are you intending to take a further countable intersection over $n$? Could you please clarify?
                                                                                                                                                – t.b.
                                                                                                                                                Jul 4 '12 at 11:24


















                                                                                                                                              -4














                                                                                                                                              I found one beautiful application of Baire Category Theorem which is the following:



                                                                                                                                              Let $mathcal H$ be a separable Hilbert Space with countable orthonormal basis ${u_{k}}_{k=1}^{infty}$. Fix $nin mathbb N$ consider $mathrm{Span}{u_{1},u_{2},...,u_{n}}$ then the following sets are dense in $mathcal H$.



                                                                                                                                              $A_{i,j}:={uin mathcal H: (u,u_{i})neq (u,u_{j})}$ where $1leq i,j leq n$ and $ineq j$.



                                                                                                                                              Proof Hints:(a) Any proper closed vector subspace of an Hilbert Space is nowhere dense.
                                                                                                                                              (b)A closed set is nowhere dense $Leftrightarrow$ its complement is everywhere dense.






                                                                                                                                              share|cite|improve this answer























                                                                                                                                              • You mean to say that any closed proper subspace is nowhere dense.
                                                                                                                                                – Asaf Karagila
                                                                                                                                                Jul 4 '12 at 7:51










                                                                                                                                              • @ Asaf Karagila: Yes I edited
                                                                                                                                                – users31526
                                                                                                                                                Jul 4 '12 at 7:56






                                                                                                                                              • 6




                                                                                                                                                I don't understand. What you denote by $A_{i,j}$ is a finite intersection of open and dense sets: $$A_{i,j} = bigcap_{1 leq i lt j leq n} mathcal{H} smallsetminus (u_i - u_j)^perp$$ which is obviously open and dense in $mathcal{H}$ (no need for Baire here). And: what is the Span doing here? Are you intending to take a further countable intersection over $n$? Could you please clarify?
                                                                                                                                                – t.b.
                                                                                                                                                Jul 4 '12 at 11:24
















                                                                                                                                              -4












                                                                                                                                              -4








                                                                                                                                              -4






                                                                                                                                              I found one beautiful application of Baire Category Theorem which is the following:



                                                                                                                                              Let $mathcal H$ be a separable Hilbert Space with countable orthonormal basis ${u_{k}}_{k=1}^{infty}$. Fix $nin mathbb N$ consider $mathrm{Span}{u_{1},u_{2},...,u_{n}}$ then the following sets are dense in $mathcal H$.



                                                                                                                                              $A_{i,j}:={uin mathcal H: (u,u_{i})neq (u,u_{j})}$ where $1leq i,j leq n$ and $ineq j$.



                                                                                                                                              Proof Hints:(a) Any proper closed vector subspace of an Hilbert Space is nowhere dense.
                                                                                                                                              (b)A closed set is nowhere dense $Leftrightarrow$ its complement is everywhere dense.






                                                                                                                                              share|cite|improve this answer














                                                                                                                                              I found one beautiful application of Baire Category Theorem which is the following:



                                                                                                                                              Let $mathcal H$ be a separable Hilbert Space with countable orthonormal basis ${u_{k}}_{k=1}^{infty}$. Fix $nin mathbb N$ consider $mathrm{Span}{u_{1},u_{2},...,u_{n}}$ then the following sets are dense in $mathcal H$.



                                                                                                                                              $A_{i,j}:={uin mathcal H: (u,u_{i})neq (u,u_{j})}$ where $1leq i,j leq n$ and $ineq j$.



                                                                                                                                              Proof Hints:(a) Any proper closed vector subspace of an Hilbert Space is nowhere dense.
                                                                                                                                              (b)A closed set is nowhere dense $Leftrightarrow$ its complement is everywhere dense.







                                                                                                                                              share|cite|improve this answer














                                                                                                                                              share|cite|improve this answer



                                                                                                                                              share|cite|improve this answer








                                                                                                                                              edited Jul 17 '12 at 17:37


























                                                                                                                                              community wiki





                                                                                                                                              3 revs, 2 users 94%
                                                                                                                                              users31526














                                                                                                                                              • You mean to say that any closed proper subspace is nowhere dense.
                                                                                                                                                – Asaf Karagila
                                                                                                                                                Jul 4 '12 at 7:51










                                                                                                                                              • @ Asaf Karagila: Yes I edited
                                                                                                                                                – users31526
                                                                                                                                                Jul 4 '12 at 7:56






                                                                                                                                              • 6




                                                                                                                                                I don't understand. What you denote by $A_{i,j}$ is a finite intersection of open and dense sets: $$A_{i,j} = bigcap_{1 leq i lt j leq n} mathcal{H} smallsetminus (u_i - u_j)^perp$$ which is obviously open and dense in $mathcal{H}$ (no need for Baire here). And: what is the Span doing here? Are you intending to take a further countable intersection over $n$? Could you please clarify?
                                                                                                                                                – t.b.
                                                                                                                                                Jul 4 '12 at 11:24




















                                                                                                                                              • You mean to say that any closed proper subspace is nowhere dense.
                                                                                                                                                – Asaf Karagila
                                                                                                                                                Jul 4 '12 at 7:51










                                                                                                                                              • @ Asaf Karagila: Yes I edited
                                                                                                                                                – users31526
                                                                                                                                                Jul 4 '12 at 7:56






                                                                                                                                              • 6




                                                                                                                                                I don't understand. What you denote by $A_{i,j}$ is a finite intersection of open and dense sets: $$A_{i,j} = bigcap_{1 leq i lt j leq n} mathcal{H} smallsetminus (u_i - u_j)^perp$$ which is obviously open and dense in $mathcal{H}$ (no need for Baire here). And: what is the Span doing here? Are you intending to take a further countable intersection over $n$? Could you please clarify?
                                                                                                                                                – t.b.
                                                                                                                                                Jul 4 '12 at 11:24


















                                                                                                                                              You mean to say that any closed proper subspace is nowhere dense.
                                                                                                                                              – Asaf Karagila
                                                                                                                                              Jul 4 '12 at 7:51




                                                                                                                                              You mean to say that any closed proper subspace is nowhere dense.
                                                                                                                                              – Asaf Karagila
                                                                                                                                              Jul 4 '12 at 7:51












                                                                                                                                              @ Asaf Karagila: Yes I edited
                                                                                                                                              – users31526
                                                                                                                                              Jul 4 '12 at 7:56




                                                                                                                                              @ Asaf Karagila: Yes I edited
                                                                                                                                              – users31526
                                                                                                                                              Jul 4 '12 at 7:56




                                                                                                                                              6




                                                                                                                                              6




                                                                                                                                              I don't understand. What you denote by $A_{i,j}$ is a finite intersection of open and dense sets: $$A_{i,j} = bigcap_{1 leq i lt j leq n} mathcal{H} smallsetminus (u_i - u_j)^perp$$ which is obviously open and dense in $mathcal{H}$ (no need for Baire here). And: what is the Span doing here? Are you intending to take a further countable intersection over $n$? Could you please clarify?
                                                                                                                                              – t.b.
                                                                                                                                              Jul 4 '12 at 11:24






                                                                                                                                              I don't understand. What you denote by $A_{i,j}$ is a finite intersection of open and dense sets: $$A_{i,j} = bigcap_{1 leq i lt j leq n} mathcal{H} smallsetminus (u_i - u_j)^perp$$ which is obviously open and dense in $mathcal{H}$ (no need for Baire here). And: what is the Span doing here? Are you intending to take a further countable intersection over $n$? Could you please clarify?
                                                                                                                                              – t.b.
                                                                                                                                              Jul 4 '12 at 11:24







                                                                                                                                              protected by user99914 Dec 17 '17 at 21:17



                                                                                                                                              Thank you for your interest in this question.
                                                                                                                                              Because it has attracted low-quality or spam answers that had to be removed, posting an answer now requires 10 reputation on this site (the association bonus does not count).



                                                                                                                                              Would you like to answer one of these unanswered questions instead?



                                                                                                                                              Popular posts from this blog

                                                                                                                                              Human spaceflight

                                                                                                                                              Can not write log (Is /dev/pts mounted?) - openpty in Ubuntu-on-Windows?

                                                                                                                                              File:DeusFollowingSea.jpg